Re: [obm-l] Irredutíveis e Anéis

2005-05-11 Por tôpico Claudio Buffara
on 11.05.05 17:12, Chicao Valadares at [EMAIL PROTECTED] wrote:

 estou meio enferrujado, nao sei se esta certo mas ai
 vai:
 
 Z[sqrt3] isomorfo a Z[x]/(x^2 - 3)
 = Z[sqrt3]/(p) isomorfo a Z[x]/(x^2 - 3)/(p)

Oi, Chicao:

Vou escrever os detalhes pra guardar esta solucao no meu arquivos.

O p do lado esquerdo eh um elemento de Z[raiz(3)] e o p do lado direito um
elemento de Z[x]/x^2-3.
Assim, um elemento tipico do ideal p da esquerda eh:
(a + b*raiz(3))*p (a, b em Z)
enquanto que do ideal p da direita eh:
(a + b*x + x^2-3)*(p + x^2-3) = (a + b*x)*p + x^2-3

Como os dois aneis sao isomorfos, voce pode fazer a identificacao.
O isomorfismo original leva a + b*raiz(3)  em  a + b*x + x^2-3.

Nos aneis quociente, ele leva:
a + b*raiz(3) + p  em  a + b*x + x^2-3 + p = a + b*x + x^2-3,p.
 
 = Z[sqrt3]/(p) isomorfo a Z[x]/(x^2 - 3 ,p)

Elemento tipico do anel quociente da esquerda:
a + b*raiz(3) + p, com a, b em Z.

Elemento tipico do anel quociente da direita:
a + b*x + x^2-3,p, onde 0 = a,b = p-1.

Isso quer dizer que estes aneis quociente tem p^2 elementos cada.


 = Z[sqrt3]/(p) isomorfo a Z[x]/(p)/(x^2 - 3 ,p)/(p)

Esse eh o 3o. teorema dos isomorfismos. Foi nesse ponto que eu empaquei,
pois nao me lembrei dele. Boa sacada!

Z[x]/p ~ (Z/p)[x] ~ Z_p[x].
x^2 - 3,p/p ~ x^2 - 3.
Logo:

 = Z[sqrt3]/(p) isomorfo a Zp[x]/(x^2 - 3)

 Desde que x^2 -3 é irredutivel=primo(hipotese) em
 Zp[x](Zp[x] é DFU) =Zp[x]/(x^2 - 3) é anel de
 integridade = 
 Z[sqrt3]/(p) é anel de integridade = (p) é primo em
 Z[sqrt3].
 
Excelente! Muito obrigado.

[]s,
Claudio.



=
Instruções para entrar na lista, sair da lista e usar a lista em
http://www.mat.puc-rio.br/~nicolau/olimp/obm-l.html
=


Re: [obm-l] Irredutíveis e Anéis

2005-05-11 Por tôpico Claudio Buffara
Oi, Gugu:

Obrigado pela solucao.

[]s,
Claudio.

on 11.05.05 15:03, Carlos Gustavo Tamm de Araujo Moreira at [EMAIL PROTECTED]
wrote:

 Se p divide (a+b.raiz(3))(c+d.raiz(3)), entao p divide
 (a^2-3b^2)(c^2-3d^2), e logo p divide um desses fatores, digamos a^2-3b^2.
 Como x^2-3 'e irredutivel, e logo nao tem raiz em Z/pZ, se p divide a^2-3b^2
 entao p divide b (senao b e' invertivel em Z/pZ, e a/b e' raiz de x^2-3), e
 logo p divide a, donde p divide a+b.raiz(3).
 Abracos,
 Gugu
 
 

=
Instruções para entrar na lista, sair da lista e usar a lista em
http://www.mat.puc-rio.br/~nicolau/olimp/obm-l.html
=


[obm-l] Isomorfismos de Grupos

2005-05-06 Por tôpico Claudio Buffara
Sejam:
R = conjunto dos numeros reais;
Q = conjunto dos numeros racionais;
A = conjunto dos numeros algebricos reais (reais que sao raizes de algum
polinomio com coeficientes inteiros);
X+ = conjunto dos elementos positivos de X (X = R, Q ou A).

Sabemos que os grupos (R,+) e (R+,*) sao isomorfos (soma e produto usuais de
numeros reais). Um isomorfismo eh, por exemplo, a funcao exponencial.

Uma pergunta interessante eh: existe algum isomorfismo entre estes grupos
que nao seja uma funcao do tipo f(x) = a^x, com a positivo e  1?

A resposta (negativa) eh dada pela solucao do seguinte problema, que jah
apareceu aqui na lista ha tempos, mas como recordar eh viver...:
Seja f uma funcao real tal que f(0) = 1, f(1) = a  0 e, para quaisquer x e
y reais, f(x+y) = f(x)*f(y).
1) Prove que, para todo racional r, f(r) = a^r;
2) Prove que f eh continua;
3) Prove que f eh diferenciavel;
4) Conclua que f(x) = a^x, para todo x real.
 
Voltando aos isomorfismos, nao eh dificil mostrar que (Q,+) e (Q+,*) nao sao
isomorfos. (dica: se f eh um isomorfismo e f(a) = 2, quem eh f(a/2)?)

O problema acima (mais precisamente, o item 1) tem um corolario
interessante, que nao pode ser demonstrado apenas com o argumento simples
usado no caso dos racionais: (A,+) nao eh isomorfo a (A+,*).
(de fato, eu acho que precisa usar o teorema de Gelfond-Schneider: se a eh
um algebrico diferente de 0 e 1 e b eh um algebrico irracional, entao a^b eh
transcendente).

Alias, um bom exercicio eh provar que estes dois grupos sao realmente
grupos, ou seja, que a soma de dois algebricos reais eh um algebrico real e
o produto de dois algebricos reais positivos eh um algebrico real positivo.

[]s,
Claudio.

=
Instruções para entrar na lista, sair da lista e usar a lista em
http://www.mat.puc-rio.br/~nicolau/olimp/obm-l.html
=


[obm-l] Mais Isomorfismos

2005-05-06 Por tôpico Claudio Buffara
Uma duvida: o grupo aditivo dos reais eh isomorfo ao grupo aditivo dos
complexos?

Nao eh dificil ver que os grupos multiplicativos dos reais nao nulos e dos
complexos nao nulos nao sao isomorfos (dica: o problema estah no i), mas no
caso dos grupos aditivos, o fato de i^2 = -1 nao parece ter nenhuma
importancia.

Por exemplo, se f:C - R for um isomorfismo, com f(1) = a e f(i) = b, entao:
f(r) = r*a e f(r*i) = r*b para cada racional r.
Logo, para quaisquer racionais r e s, f(r + s*i) = r*a + s*b.
Isso implica que a e b devem ser LI sobre Q pois, caso contrario, f nao
seria injetiva.

Talvez esse problema esteja relacionado a existencia de uma funcao g:R - R
que satisfaz a g(x+y) = g(x)+g(y) para quaisquer x e y reais mas que eh
descontinua em todo ponto (lah vem o axioma da escolha de novo...)

[]s,
Claudio.


=
Instruções para entrar na lista, sair da lista e usar a lista em
http://www.mat.puc-rio.br/~nicolau/olimp/obm-l.html
=


Re: [obm-l] Mais Isomorfismos

2005-05-06 Por tôpico Claudio Buffara
on 06.05.05 17:22, Nicolau C. Saldanha at [EMAIL PROTECTED] wrote:

 On Fri, May 06, 2005 at 04:12:43PM -0300, Claudio Buffara wrote:
 Uma duvida: o grupo aditivo dos reais eh isomorfo ao grupo aditivo dos
 complexos?
 
 Sim, ambos são Q-espaços vetoriais de mesma dimensão (card(R)).
 
 []s, N.

OK. Obrigado.

Como eh que se demonstra que dois espacos vetoriais sobre o mesmo corpo com
mesma dimensao infinita sao isomorfos? Onde entra o axioma da escolha? Soh
na existencia das bases?

[]s,
Claudio.


=
Instruções para entrar na lista, sair da lista e usar a lista em
http://www.mat.puc-rio.br/~nicolau/olimp/obm-l.html
=


Re: [obm-l] Olimpiada Relampago

2005-05-05 Por tôpico Claudio Buffara
on 04.05.05 11:02, Nicolau C. Saldanha at [EMAIL PROTECTED] wrote:

 No dia 15 de abril houve aqui na PUC um evento chamado PUC por um dia.
 Neste dia eu organizei uma olimpíada relâmpago, com alguns dos meus
 problemas olímpicos mas relativamente fáceis favoritos. Convido vocês
 a darem uma olhada. Está aqui:
 
 http://www.mat.puc-rio.br/~nicolau/olimp/20050415/
 
 []s, N.

O problema 4 tem uma variante interessante: achar um real positivo a tal que
[a^n] tem a mesma paridade de n.

[]s,
Claudio.


=
Instruções para entrar na lista, sair da lista e usar a lista em
http://www.mat.puc-rio.br/~nicolau/olimp/obm-l.html
=


[obm-l] Anel com 2005 irredutiveis

2005-05-05 Por tôpico Claudio Buffara
Esse pode servir de preparacao pra obm-u (1a. fase, claro!):

De um exemplo de um anel que tem precisamente 2005 elementos irredutiveis.
Quantos ideais primos tem esse anel?
Quantos ideais maximais?

Voce consegue dar um exemplo com caracteristica  0?

[]s,
Claudio.

=
Instruções para entrar na lista, sair da lista e usar a lista em
http://www.mat.puc-rio.br/~nicolau/olimp/obm-l.html
=


[obm-l] Nao Grupo

2005-05-05 Por tôpico Claudio Buffara
Alguem conhece algum exemplo de um conjunto S, fechado em relacao a uma
operacao associativa * e tal que:
1) Existe e em S tal que a*e = a, para todo a em S;
2) Para todo a em S, existe b em S tal que b*a = e;
3) S NAO eh um grupo ?

[]s,
Claudio.

=
Instruções para entrar na lista, sair da lista e usar a lista em
http://www.mat.puc-rio.br/~nicolau/olimp/obm-l.html
=


Re: [obm-l] autovalores , autovetores

2005-05-01 Por tôpico Claudio Buffara
on 29.04.05 17:56, Fabio Niski at [EMAIL PROTECTED] wrote:

 Obrigado Claudio.
 Alias, sobre a sua afirmativa u*u'tem posto 1 e, portanto, n-1
 autovalores são iguais a 0.  veja, por gentileza, se o meu argumento
 esta correto:
 
 Como A é simetrica podemos escreve-la da seguinte maneira
 A = c1*e1'*e1 + ... + cn*en'*en
 onde os ci sao os autovalores e os ei os autovetores correspondentes.
 Como A tem posto 1 e os autovalores sao l.i só se pode ter um autovalor
 diferente de 0.

Essa expressao para A nao me parece obvia a priori.
Alem disso, como voce prova que os autovetores (nao autovalores) sao L.I.?
Eles soh serao L.I. a priori se os autovalores correspondentes forem
distintos, o que nao ocorre nesse caso (n-1 deles sao iguais a 0).

 Voce provaria de outra maneira?

O nucleo de A eh um subespaco de R^n.
Posto(A) = 1 == dim(Nucleo(A)) = n-1 (teorema do nucleo e da imagem)
Logo, Nucleo(A) tem uma base (de fato, uma infinidade de bases) com n-1
vetores.
Para qualquer vetor v dessa base (de fato, qualquer vetor do nucleo), vale
Av = 0 = 0v, ou seja, cada vetor da base eh um autovetor associado ao
autovalor 0.
Como a base tem n-1 vetores L.I., o autovalor 0 tem multiplicidade = n-1.
O n-esimo autovetor eh u (ou qualquer multiplo escalar nao nulo de u), com
autovalor associado igual a |u|^2  0 (a menos que u = 0, mas nesse caso A
seria a matriz nula).
Logo, a multiplicidade do autovalor 0 eh n-1.
Se A = 0, entao 0 eh autovalor de multiplicidade n e o auto-espaco associado
eh todo o R^n.
 
[]s,
Claudio.

 Abraços
 
 
 claudio.buffara wrote:
 
 Oi, Niski:
 
 Estou supondo que u é um vetor coluna do R^n.
 Nesse caso, a matriz u*u' tem o elemento (i,j) igual a u(i)*u(j)
 (produto da i-ésima e j-ésima componentes de u).
 Ou seja, a i-ésima linha de u*u' é igual a u(i)*u.
 Logo, u*u' tem posto 1 e, portanto, n-1 autovalores são iguais a 0.
 Multiplicando u*u' por u, obtemos (u(1)^2 + ... + u(n)^2)*u.
 Logo, u é autovetor com o autovalor associado igual a:
 u(1)^2 + ... + u(n)^2 = |u|^2.
 
 []s,
 Claudio.
 
 De:  [EMAIL PROTECTED]
 
 Para:  obm-l@mat.puc-rio.br
 
 Cópia:  
 
 Data:  Fri, 29 Apr 2005 15:22:27 -0300
 
 Assunto:  [obm-l] autovalores , autovetores
 
 Pessoal, como eu resolvo este problema:
 
 Encontre os autovalores e autovetores de uma matriz A = u.u', onde u
 pert R^n
 (notacao: u' = u transposto)
 
 Sem precisar recorer a resolver equacoes genericas escabrosas (isto é
 sem recorrer ao artificio das raizes da eq. det|A - cI| = 0)
 
 
 Obrigado.


=
Instruções para entrar na lista, sair da lista e usar a lista em
http://www.mat.puc-rio.br/~nicolau/olimp/obm-l.html
=


Re: [obm-l] Área entre curvas

2005-04-30 Por tôpico Claudio Buffara
on 30.04.05 13:57, [EMAIL PROTECTED] at [EMAIL PROTECTED] wrote:

 Seja C uma curva plana convexa e fechada (de classe C^1). Considere um
 segmento que desliza sobre C (com extremidades em C e comprimento fixo) até
 dar uma volta completa. Considere a curva K descrita por um ponto P do
 segmento, situado a distândias a e b das extremidades. Mostre que a área da
 região compreendida entre C e K é pi*a*b.
 
 []s,
 Daniel
 
Legal esse problema. Aqui vai minha tentativa de solucao heuristica.

Se C for uma circunferencia, a demonstracao sai facil usando a potencia de P
em relacao a C. Naturalmente, P irah descrever uma circunferencia de raio d
concentrica com C, cujo raio eh r. Chamando o segmento de XY e o diametro
contendo P de AB, teremos: |XP|*|PY| = |AP|*|PB| == a*b = (r-d)*(r+d) ==
r^2-d^2 = a*b == Area Desejada = pi*(r^2 - d^2) = pi*a*b.

Em particular, tomando um elemento de area dS, correspondente ao setor
circular de C subtendido por um angulo dt, teremos que:
dS = (1/2)*(r^2-d^2)*dt = (1/2)*a*b*dt.

No caso geral, como C eh uma curva plana convexa fechada de classe C^1, ela
eh localmente uma circunferencia (no sentido de que, para efeitos de calculo
de curvatura e area, podemos desprezar os termos de ordem = 3), de modo
que, no arco de C delimitado pelo segmento, vai existir um ponto A tal que
A, P e O sao colineares (O = centro de curvatura relativo ao ponto A).

A medida que o segmento desliza, o ponto A varia continuamente (pois C eh de
classe C^1) e, apos uma volta completa (2pi radianos) do vetor curvatura
(ligando A a O) volta a posicao original (pois C eh fechada). Alem disso, P
ficarah sempre entre A e O (pois C eh convexa), de modo que o integrando
(elemento de area) nunca muda de sinal, permanecendo sempre positivo.

Dai, usando o resultado estabelecido pra circunferencias, achamos que a area
desejada eh igual a:
Integral(0...2pi) (1/2)*(r^2 - d^2)*dt = (1/2)*2*pi*a*b = pi*a*b.
  
Agora, eh soh formalizar essa baboseira que eu escrevi acima.


[]s,
Claudio.


=
Instruções para entrar na lista, sair da lista e usar a lista em
http://www.mat.puc-rio.br/~nicolau/olimp/obm-l.html
=


Re: [obm-l] Problema 1 da XXV OBM - Nível 1, fase 3

2005-04-29 Por tôpico Claudio Buffara
O que ele fez foi calcular o numero de solucoes inteiras e nao-negativas de
x + y + z = 7, onde x eh o algarismo das centenas, y o das dezenas e z o das
unidades. Isso eh igual a Binom(7+3-1,3-1) = Binom(9,2) = 36.

Mas o problema eh facil de fazer no braco:

Combinacoes de algarismos que somam 7:
0,0,7 == 3 solucoes (7, 70 e 700)
0,1,6 == 6 solucoes (16, 61, 106, 160, 601 e 610)
0,2,5 == 6
0,3,4 == 6
1,1,5 == 3
1,2,4 == 6
1,3,3 == 3
2,2,3 == 3
Total: 36 solucoes.

***

E o que eh C(7,9)?

[]s,
Claudio.

on 29.04.05 11:05, Eduardo Wilner at [EMAIL PROTECTED] wrote:

 Prezado João Carlos
 
 Poderia explicar melhor tua solução?
 Parece que vc. chega a C(7,9)! De onde?
 Porque os algarismos resultam como restos da divisão
 por 7?
 Eu encontrei 42 ...!
 
 Abraço
 Wilner
 
 
 --- [EMAIL PROTECTED] wrote:
 Como sempre gentil, obrigado: amigo Buffara.
 
 
 
 
 Claudio Buffara [EMAIL PROTECTED]
 Enviado Por: [EMAIL PROTECTED]
 28/04/2005 21:22
 Favor responder a obm-l
 
 Para:   obm-l@mat.puc-rio.br
 cc: 
 Assunto:Re: [obm-l] Problema 1 da
 XXV OBM - Nível 1, fase
 3
 
 
 Sim.
 
 on 28.04.05 19:46, [EMAIL PROTECTED]
 at 
 [EMAIL PROTECTED] wrote:
 
 Quantos inteiros positivos menores que 1.000 têm
 soma de seus algarismos
 igual a 7?
 Pergunta: essa solução que segue abaixo faz sentido?
 Solução: esse problema é equivalente a encontrar o
 número de soluções
 inteiras para a equação: x+y+z=7, na qual x, y e z
 são os restos da
 divisão da centena, dezena e unidade do inteiro
 (menor que 1000) por 7, ou
 seja, 9!/(7!2!)=36.
 
 
 


=
Instruções para entrar na lista, sair da lista e usar a lista em
http://www.mat.puc-rio.br/~nicolau/olimp/obm-l.html
=


Re: [obm-l] VAlor mínimo

2005-04-28 Por tôpico Claudio Buffara
Title: Re: [obm-l] VAlor mínimo



Expresse y em funcao de x e substitua na expressao pra z.
z serah uma funcao quadratica de x cujo minimo eh facil de calcular.

on 28.04.05 20:28, Robÿe9rio Alves at [EMAIL PROTECTED] wrote:

Se x e y são reais tais que 3x + 4y = 12, determinar o valor mínimo de z = x^2 + y^2 .

Yahoo! Acesso Grátis  http://us.rd.yahoo.com/mail/br/taglines/*http://br.acesso.yahoo.com// : Internet rápida e grátis. Instale o discador agora! 






Re: [obm-l] Problema 1 da XXV OBM - Nível 1, fase 3

2005-04-28 Por tôpico Claudio Buffara
Title: Re: [obm-l] Problema 1 da XXV OBM - Nível 1, fase 3



Sim.

on 28.04.05 19:46, [EMAIL PROTECTED] at [EMAIL PROTECTED] wrote:

Quantos inteiros positivos menores que 1.000 têm soma de seus algarismos igual a 7?
Pergunta: essa solução que segue abaixo faz sentido?
Solução: esse problema é equivalente a encontrar o número de soluções inteiras para a equação: x+y+z=7, na qual x, y e z são os restos da divisão da centena, dezena e unidade do inteiro (menor que 1000) por 7, ou seja, 9!/(7!2!)=36.






Re: [obm-l] Elementos de um Grupo

2005-04-28 Por tôpico Claudio Buffara
on 28.04.05 18:23, Chicao Valadares at [EMAIL PROTECTED] wrote:

 donde b*(a*b)^9=(b*a)^9*b???
 
 
 --- Carlos Gustavo Tamm de Araujo Moreira
 [EMAIL PROTECTED] wrote:
 Oi Cláudio,

 De a^(-1)*b^2*a = b^3 segue b^2*a*b^(-2)=a*b

e portanto b^2*a^9*b^(-2) = (a*b)^9

 De b^(-1)*a^2*b = a^3 segue

b^(-1)*a^4*b = a^6   e   b^(-1)*a^6*b = a^9 ==

 b^(-2)*a^4*b^2 = b^(-1)*a^6*b = a^9, donde
 a^4 = b^2*a^9*b^(-2) = (a*b)^9.

 Analogamente, b^4=(b*a)^9.

 Assim,
 b*a^4 = b*(a*b)^9 = (b*a)^9*b = b^4*a, donde
 a^3 = b^3, e de a^(-1)*b^2*a = b^3 = a^3 segue
 b^2 = a^3 = b^3, donde b = e, e
 analogamente a = e.
 Abraços,
 Gugu
 
 
 a e b sao elementos de um grupo e satisfazem a:
 a^(-1)*b^2*a = b^3   e   b^(-1)*a^2*b = a^3
 Prove que a = b = e = identidade do grupo.
 
 []s,
 Claudio.
 


=
Instruções para entrar na lista, sair da lista e usar a lista em
http://www.mat.puc-rio.br/~nicolau/olimp/obm-l.html
=


[obm-l] grupo de matrizes

2005-04-28 Por tôpico Claudio Buffara
Noutro dia, foi provado na lista que o conjunto (de fato, o grupo) GL(n,R)
das matrizes invertiveis nxn com coeficientes reais eh aberto em R^(nxn).
Foi soh observar que GL(n,R) eh a imagem inversa do aberto R - {0} da reta
pela funcao continua determinante.

Que tal esse aqui?

Seja G um grupo multiplicativo de matrizes reais nxn.
Prove que se G tem interior nao-vazio entao G eh aberto.

[]s,
Claudio.

=
Instruções para entrar na lista, sair da lista e usar a lista em
http://www.mat.puc-rio.br/~nicolau/olimp/obm-l.html
=


Re: [obm-l] Tetei muito

2005-04-24 Por tôpico Claudio Buffara
Title: Re: [obm-l] Tetei muito



Sem supor que x e y sao positivos (ou, pelo menos limitados inferiormente), ax + by fica ilimitado inferiormente e, portanto, nao atinge um valor minimo.

Pra ver isso, tome M positivo e arbitrariamente grande e x = -M/a.
Dai, y = -ac/M e ax + by = -M - abc/M  -M.

[]s,
Claudio.

on 23.04.05 19:24, Robÿe9rio Alves at [EMAIL PROTECTED] wrote:
 
06) Dados a, b e c positivos, determinar x e y tais que xy = c e que ax + by seja o menor possível. 







[obm-l] Elementos de um Grupo

2005-04-23 Por tôpico Claudio Buffara
a e b sao elementos de um grupo e satisfazem a:
a^(-1)*b^2*a = b^3   e   b^(-1)*a^2*b = a^3
Prove que a = b = e = identidade do grupo.

[]s,
Claudio.

=
Instruções para entrar na lista, sair da lista e usar a lista em
http://www.mat.puc-rio.br/~nicolau/olimp/obm-l.html
=


[obm-l] Re: Subespaco Vetorial Fechado

2005-04-23 Por tôpico Claudio Buffara
Title: Re: Subespaco Vetorial Fechado 



Acho que uma prova mais topologica vai depender da topologia que voce definir, ou seja, quem sao os subconjuntos abertos de E? (os fechados serao justamente aqueles cujo complementar eh aberto)

No caso do R^n e do livro do Elon, um subconjunto A eh aberto se, por definicao, para cada ponto de A, existe uma bola aberta que contem o tal ponto e estah contida em A.

[]s,
Claudio.

on 23.04.05 19:11, Bruno Lima at [EMAIL PROTECTED] wrote:

No livro do Elon Analise 2 . cap1. Tem um problema: (*)Seja F subspaco vetorial de R^n , mostre que F é fechado. As provas que vi todas usam o fato do ambiente ter dim finita, ie, tome uma base...
Eu nao sei nada de Analise Funcional , mas parece(intuiçao) que isso tambem vale com dimensao infinita.
Alguem ai saberia um contra-exemplo em dimensao infnita ou uma prova do fato (*) que nao use base ou coisas equivalentes, quero dizer uma prova mais topológica
 
Valeu.

Yahoo! Acesso Grátis  http://us.rd.yahoo.com/mail/br/taglines/*http://br.acesso.yahoo.com// : Internet rápida e grátis. Instale o discador agora! 






[obm-l] Fatoracao Unica em D[x]

2005-04-20 Por tôpico Claudio Buffara
Estou empacado neste aqui:

Seja D um dominio de integridade que nao tem fatoracao unica.

Prove ou de um contra-exemplo:
Todo polinomio monico de D[x] pode ser expresso, de forma unica, como o
produto de polinomios irredutiveis em D[x].

Eu sei que se F eh o corpo de fracoes de D, entao qualquer f(x) de D[x] tem
fatoracao unica em F[x]. O problema eh que o lema de Gauss usa os tais
polinomios primitivos, onde o mdc dos coeficientes eh 1, soh que se D nao eh
fatorial, nao existe mdc...

[]s,
Claudio.

=
Instruções para entrar na lista, sair da lista e usar a lista em
http://www.mat.puc-rio.br/~nicolau/olimp/obm-l.html
=


[obm-l] Pontos de Inflexao

2005-04-10 Por tôpico Claudio Buffara
Aqui vai um resultado curioso:

Seja p(x) um polinomio de grau 4 com dois pontos de inflexao, cujas
abscissas sao i1 e i2 com i1  i2.
Seja r a reta que passa por estes dois pontos.
Prove que esta reta intersecta o grafico de p(x) em dois outros pontos, de
abscissas x1 e x2 tais que x1  i1  i2  x2 e que:
(i2-i1)/(i1-x1) = (i2-i1)/(x2-i2) = K, onde K eh uma constante que independe
de p(x) (desde que este tenha dois pontos de inflexao).
Quanto vale K?

Dica: este eh um daqueles problemas onde uma mudanca de coordenadas adequada
ajuda muito. Ou seja, s.p.d.g. voce pode supor que p(x) tem uma forma muito
simples.

[]s,
Claudio.

=
Instruções para entrar na lista, sair da lista e usar a lista em
http://www.mat.puc-rio.br/~nicolau/olimp/obm-l.html
=


Re: [obm-l] Por 7!!!(???) DE NOVO!

2005-04-10 Por tôpico Claudio Buffara
Mod 7:
1 == 1
10 == 3
100 == 2 == 
(abc) = 100a + 10b + c == 2a + 3b + c (mod 7)

Logo, 7 divide (abc) == 7 divide 2a + 3b + c

1000 == -1
1 == -3
10 == -2 == 
(abcdef) = 10a + 1b + 1000c + 100d + 10e + f ==
-2a -3b -c + 2d + 3e + f == -(2a+3b+c) + (2d+3e+f) (mod 7)

Logo, 7 divide (abcdef) == 7 divide -(2a+3b+c) + (2d+3e+f)

E por ai vai

Ficou claro?

Entao farelo pra voce tambem.

[]s,
Claudio.
 
on 10.04.05 12:10, Sinomar Dias at [EMAIL PROTECTED] wrote:

 
 
 Colegas, já que ninguém quis me ajudar no problema, poderiam me dizer onde
 encontrar uma demonstração para o seguinte fato relativo ao  critério  de
 divisibilidade por 7, como está descrito abaixo?
 Obrigado por qualquer ajudinha.
 
 
 i) Um número natural n de 3 ou menos algarismos é divisível por 7 se
 ocorrer o que segue:
 
 Dadon=abc ( a,b e c são os algarismos do número) se, 2*a+3*b+c é
 divisível por 7, então n é divisível por  7.
 
 ii) Um natural n com mais de  3 algarismos é divisível por  7 se, separado
 em classes de 3 algarismos a partir do último (inclusive), a diferença
 entre a soma das classes de ordem ímpar e de ordem par for um número
 divisível por 7, independente do sinal:
 
 Dado n=abcdefg
 
 Classe1: efg
 Classe2: bcd
 Classe3: a
 
 S(I)=efg+a ( soma das classes de ordem ímpar)
 S(P)=bcd (soma das classes de ordem par)
 
 Se S(I) – S(P) for divisível por 7, então n é divisível por 7.
 
 Obrigado
 
 Farelo!!!
 


=
Instruções para entrar na lista, sair da lista e usar a lista em
http://www.mat.puc-rio.br/~nicolau/olimp/obm-l.html
=


Re: [obm-l] Sistemas Dinâmicos

2005-04-09 Por tôpico Claudio Buffara
Title: Re: [obm-l] Sistemas Dinâmicos



Nao entendi muito bem como voce pode apertar SIMULTANEAMENTE as teclas sen e cos da calculadora e obter algum resultado que nao seja Error.

Interpretando o que voce quis dizer duma forma que me parece razoavel, eu vejo tres casos:
1) Se voce soh apertar sen, a sequencia converge pra zero.

2) Se voce soh apertar cos, a sequencia converge pra a tal que a = cos(a).

3) Se voce apertar alternadamente sen e cos, voce obterah uma sequencia com duas subsequencias convergentes intercaladas, uma das quais converge para b = sen(cos(b)) e a outra para c = cos(sen(c)). Nesse caso, voce pode trata-las como sequencias independentes, uma formada pelos termos de ordem impar e a outra pelos termos de ordem par da sequencia original.

O primeiro caso eh o mais facil, pois (a_n) eh claramente monotona e limitada. Logo, converge para a tal que a = sen(a) == a = 0.

Uma demonstracao dos casos 2 e 3 pode ser baseada no seguinte teorema:
Sejam I um intervalo e f:I - R uma funcao diferenciavel no interior de I.
Se existe uma constante real k tal que, para todo x em int(I), |f'(x)| = k  1, entao 
a sequencia (a_n) dada por a_n = f(a_(n-1)) converge para um limite a tal que a = f(a), qualquer que seja o valor de a_0 pertencente a I.

Pra provar o teorema (um exercicio que vale a pena), use o TVM e a desigualdade triangular. Voce vai manipular e somar desigualdades, somar uma PG (razao k) e concluir que a sequencia eh de Cauchy e, portanto, convergente. A continuidade de f garante que a = f(a).
 
Em todos os casos, tome I = [-1,1] e aplique o lema a partir do segundo termo da sequencia (que pertence necessariamente a este intervalo).
Pro segundo caso, use f(x) = cos(x) e pro terceiro, f(x) = sen(cos(x)) e g(x) = cos(sen(x)).

[]s,
Claudio.

on 09.04.05 21:33, Ronaldo Luiz Alonso at [EMAIL PROTECTED] wrote:

 Este problema me foi proposto quando estava no
colegial. Hoje sei como resolver, mas na época era
enigmático. De qualquer maneira
costuma aparecer em olimpíadas e vale 
a pena lançá-lo nesta lista
para as pessoas tomarem ciência dele.
 
 --
 Uma pessoa digita um número qualquer na calculadora
e em seguida aperta simultâneamente as teclas sen
e cos sem parar. 
 a) A sequência converge?
 b) Qual número teremos no final?
 
 
PS: faça essa experiência em seu computador. :) 
[]s a todos.







Re: [obm-l] Re: [obm-l] Re: [obm-l] cálculo no R^n

2005-04-08 Por tôpico Claudio Buffara
Injetiva:
f(x) = f(y) == x,xx = y,yy.
Se x = 0, entao y,yy = 0 e isso se e soh se y = 0.
Se x  0, entao x,x  0 e x = y,y/x,xy.
y nao pode ser 0, pois nesse caso teriamos x = 0, uma contradicao.
Logo, y,y  0 e x = ky, onde k = y,y/x,x  0.
Assim, x,x = ky,ky = k^2y,y ==
1/k^2 = y,y/x,x = k ==
k^3 = 1 == 
k = 1, pois k eh real ==
x = y ==
f eh injetiva.

Mesmo em R^1 a inversa nao eh diferenciavel, pois nesse caso f(x) = x^3 e a
inversa g(x) = x^(1/3) nao eh diferenciavel na origem.

Seja g: R^n - R^n a inversa de f.
Entao, g(y) = y/y,y^(1/3) se y  0  e  g(0) = 0. Pode fazer as contas.

Se g for diferenciavel na origem, vai existir uma transformacao linear T tal
que:
g(h) = g(0) + T*h + r(h), tal que r(h)/|h| - 0 quando h - 0  ==
r(h) = h/h,h^(1/3) - T*h.
Tome h da forma k*e_1, onde k eh real e e_1 = (1,0,0,...,0).
Entao, h/h,h^(1/3) = k^(1/3)*e_1  e  T*h = k*(t_1,t_2,...,t_n), onde os
t_i dependem de T.
Logo, r(h) = (k^(1/3) - k*t_1,-k*t_2,...,-k*t_n).

|h| = raiz(h,h) = |k| ==
r(h)/|h| = (k^(1/3)/|k| - kt_1/|k|,-kt_2/|k|,...,-kt_n/|k|).
Quando k - 0 (e portanto |h| - 0), as coordenadas 2, 3, ..., n soh terao
limite se t_2 = t_3 = ... = t_n = 0.
Mesmo nesse caso, k^(1/3)/|k| - kt_1/|k| eh ilimitada numa vizinhaca de
zero, de modo que r(h)/|h| nao tende a zero. Ou seja, g nao eh diferenciavel
na origem.

[]s,
Claudio.

on 08.04.05 01:43, Ronaldo Luiz Alonso at [EMAIL PROTECTED]
wrote:

 Meu caro Ronaldo,
 acho que seu argumento que f é uma contração na bola
 B(0,1) não está correta, pois não por enquanto não
 temos uma constante 0 = k  1 tal que ||f(x) - f(y)||
 = k.||x - y||. Apesar de mesmo aceitando esse
 hipótese, também não fiquei convensido que ela
 injetiva e não adimite inversa diferenciável!!
 Sem mais.
 
 Acho que você como matemático está certo em
 julgamento.   De fato, matemáticos querem
 sempre coisas  precisas.  A intuição ajuda muito
 mas não convence  :)
 
 Deixa-me tentar novamente:
 Acredito que a constante k pode ser obtida pela
 desigualdade triangular.
 ||f(x) + (- f(y))|| = ||f(x)|| + ||-f(y)||  = ||x,xx|| +
 ||y,yy|| = ||x||^2.||x|| + ||y||^2.||y|| = ||x||^3 + ||y||^3
 
 como ||x||1 e ||y||  1, então ||x||^3+||y||^3  ||x||+||y||
 ||x|| - ||y|| (pois a norma é sempre positiva).
 então qualquer 0 = k  1 satisfaz a desigualdade.
 
 Está certo?
 
 Falta tempo para eu examinar melhor as
 idéias (e talvez também competência minha,
 para firmá-las).
 []s e saudações.
 
 
 --- Ronaldo Luiz Alonso
 [EMAIL PROTECTED] wrote:
 -
 2) Seja f: R^n -- R^n dada por f(x) = x,x.x.
 Mostre que f é de classe C infinito e que leva a
 bola unitária B(0;1) sobre si mesma injetivamente.
 Mostre que, entretanto, a aplicação inversa não é
 diferenciável na origem.
 
 Neste caso se x \in B(0;1) então x,x = ||x|| e
 0||x||  1.   Logo a aplicação é uma contração de
 x.
 A contração é diferenciável e de classe
 C^{\infty}.
 É mais ou menos intuitivo que neste caso a apliação
 seja
 injetiva.  Por exemplo: Vetores próximos da
 fronteira
 tem norma 1 e portanto serão pouco contraídos.
 Assim a demonstração de injetividade usa esse
 fato,
 isto é, se tomarmos um ponto x próximo próximo da
 fronteira, podemos sempre escolher um f^{-1}(x) tal
 que f composto com f^{-1}(x) = x e vice versa.
 Como ||x|| é sempre   menor que 1
 esses pontos tem que ser diferentes.
 Para entender por que a aplicação não é
 diferenciável
 na origem basta notar que quanto mais perto o vetor
 estiver da origem mais contraído será na aplicação
 direta.
 (reciprocamente na aplicação inversa mais
 expandido
 será).   A origem é uma espécie de buraco
 negro ao contrário logo não pode ter derivada
 lá. Argumentos do teorema de função implícita podem
 ajudar.
 Novamente sem rigor... apenas com idéias.
 
 []s Ronaldo L. Alonso
 
 
 
 
 
 
 Yahoo! Acesso Grátis - Internet rápida e grátis.
 Instale o discador agora! http://br.acesso.yahoo.com/
 =
 Instruções para entrar na lista, sair da lista e usar a lista em
 http://www.mat.puc-rio.br/~nicolau/olimp/obm-l.html
 =
 
 =
 Instruções para entrar na lista, sair da lista e usar a lista em
 http://www.mat.puc-rio.br/~nicolau/olimp/obm-l.html
 =
 


=
Instruções para entrar na lista, sair da lista e usar a lista em
http://www.mat.puc-rio.br/~nicolau/olimp/obm-l.html
=


[obm-l] Retorno a origem

2005-04-08 Por tôpico Claudio Buffara
O problema abaixo eh uma especie de generalizacao daquele do triangulo
isosceles com um angulo de 20 graus onde aparecem varios segmentos de mesmo
tamanho:

Sao dadas duas retas r e s que se intersectam no ponto O e fazem um angulo t
uma com a outra.
Sobre uma delas (digamos r) marcamos o ponto A_1.
Depois disso, sobre s, marcamos o ponto A_2 tal que |OA_1| = |A_1A_2|.
Em seguida, sobre r, marcamos o ponto A_3 tal que A_1A_2 e A_2A_3 sao
distintos (ou seja, A_1  A_3) mas tem o mesmo comprimento.
Prosseguimos desta forma, marcando pontos sobre cada uma das retas
alternadamente, os quais formam segmentos consecutivos distintos e de mesmo
comprimento.

Determine os valores de t (em funcao de n) tais que A_n coincide com O (n
inteiro positivo = 3) mas os A_i (1=i=n) sao todos distintos de O.

[]s,
Claudio.




=
Instruções para entrar na lista, sair da lista e usar a lista em
http://www.mat.puc-rio.br/~nicolau/olimp/obm-l.html
=


Re: [obm-l] soma de termos

2005-04-07 Por tôpico Claudio Buffara
on 07.04.05 10:28, Nicolau C. Saldanha at [EMAIL PROTECTED] wrote:

 On Wed, Apr 06, 2005 at 03:58:30PM -0300, claudio.buffara wrote:
 Por exemplo, é possível dar uma demonstração combinatória da identidade
 abaixo, que foi uma questão da famosa e difícil prova do IME de 1980/81.
 
 SOMA(k=0...n) Binom(k,m)*Binom(n-k,m) = Binom(n+1,2m+1).
 
 Agora, quero ver alguém provar isso algebricamente...
 
 O fato (que não é difícil) que você deve conhecer para fazer isto
 algebricamente é que f_m(x) = x^m/(1-x)^(m+1) = soma_k binom(k,m) x^k.

Eu nao conhecia essa identidade e, francamente, creio que nenhum
vestibulando normal teria a ideia de usa-la na hora da prova, a menos que
jah a tivesse visto antes. Enfim, como voce disse, nao eh dificil demonstrar
(alias, acho que o lado esquerdo deveria ser (-x)^m/(1-x)^(m+1)).

Eh soh derivar m vezes cada membro de:
1/(1-x) = SOMA(k=0) x^k  (o que vale apenas quando |x|  1)
obtendo:
m!*(-1)^m/(1-x)^(m+1) = SOMA(k=m) (k!/(k-m)!)*x^(k-m)

Multiplicando cada membro por x^m/m!, ficamos com:
(-x)^m/(1-x)^(m+1) = SOMA(k=m) Binom(k,m)*x^k.

 Assim o lado direito A é o coeficiente de x^n em
 (f_m(x))^2 = x^(2m)/(1-x)^(2m+2) = (1/x) f_(2m+1)(x).
 Portanto A é o coeficiente de x^(n+1) de f_(2m+1),
 ou seja, A = binom(n+1,2m+1).

Alias, pensando melhor, acho que somente alguem familiarizado com funcoes
geratrizes e series formais usaria isso naquela questao. Certamente, nao era
o meu caso na epoca em que prestei IME (um ano depois do seu) mas, por
sorte, a prova de MAT1 de 1981/82 foi muito mais facil...

 Mas eu concordo com o Claudio, prefiro a demonstração combinatoria.
 Alias, foi a que eu usei na prova (este foi o meu ano, e eu prestei
 o vestibular do IME, mas acabei não entrando lá).

Uma clarificacao: acabou nao entrando lah porque nao quis, pois, pra quem
nao sabe, o Nicolau foi 1o. colocado no vestibular do IME daquele ano.

[]s,
Claudio.


=
Instruções para entrar na lista, sair da lista e usar a lista em
http://www.mat.puc-rio.br/~nicolau/olimp/obm-l.html
=


Re: [obm-l] soma de termos

2005-04-07 Por tôpico Claudio Buffara
Na verdade, a formula original do Nicolau tava certa:

A m-esima derivada de 1/(1-x) eh mesmo m!/(1-x)^(m+1). O - do x cancela o
- do expoente em cada derivada sucessiva de (1-x)^(-k).

Nossa! Nao estou conseguindo nem derivar uma funcao boba dessas...acho que
tah na hora de tirar umas ferias...

[]s,
Claudio.

on 07.04.05 19:48, Claudio Buffara at [EMAIL PROTECTED] wrote:

 on 07.04.05 10:28, Nicolau C. Saldanha at [EMAIL PROTECTED] wrote:
 
 On Wed, Apr 06, 2005 at 03:58:30PM -0300, claudio.buffara wrote:
 Por exemplo, é possível dar uma demonstração combinatória da identidade
 abaixo, que foi uma questão da famosa e difícil prova do IME de 1980/81.
 
 SOMA(k=0...n) Binom(k,m)*Binom(n-k,m) = Binom(n+1,2m+1).
 
 Agora, quero ver alguém provar isso algebricamente...
 
 O fato (que não é difícil) que você deve conhecer para fazer isto
 algebricamente é que f_m(x) = x^m/(1-x)^(m+1) = soma_k binom(k,m) x^k.
 
 Eu nao conhecia essa identidade e, francamente, creio que nenhum
 vestibulando normal teria a ideia de usa-la na hora da prova, a menos que
 jah a tivesse visto antes. Enfim, como voce disse, nao eh dificil demonstrar
 (alias, acho que o lado esquerdo deveria ser (-x)^m/(1-x)^(m+1)).
 
 Eh soh derivar m vezes cada membro de:
 1/(1-x) = SOMA(k=0) x^k  (o que vale apenas quando |x|  1)
 obtendo:
 m!*(-1)^m/(1-x)^(m+1) = SOMA(k=m) (k!/(k-m)!)*x^(k-m)
 
 Multiplicando cada membro por x^m/m!, ficamos com:
 (-x)^m/(1-x)^(m+1) = SOMA(k=m) Binom(k,m)*x^k.
 
 Assim o lado direito A é o coeficiente de x^n em
 (f_m(x))^2 = x^(2m)/(1-x)^(2m+2) = (1/x) f_(2m+1)(x).
 Portanto A é o coeficiente de x^(n+1) de f_(2m+1),
 ou seja, A = binom(n+1,2m+1).
 
 Alias, pensando melhor, acho que somente alguem familiarizado com funcoes
 geratrizes e series formais usaria isso naquela questao. Certamente, nao era
 o meu caso na epoca em que prestei IME (um ano depois do seu) mas, por
 sorte, a prova de MAT1 de 1981/82 foi muito mais facil...
 
 Mas eu concordo com o Claudio, prefiro a demonstração combinatoria.
 Alias, foi a que eu usei na prova (este foi o meu ano, e eu prestei
 o vestibular do IME, mas acabei não entrando lá).
 
 Uma clarificacao: acabou nao entrando lah porque nao quis, pois, pra quem
 nao sabe, o Nicolau foi 1o. colocado no vestibular do IME daquele ano.
 
 []s,
 Claudio.
 
 
 =
 Instruções para entrar na lista, sair da lista e usar a lista em
 http://www.mat.puc-rio.br/~nicolau/olimp/obm-l.html
 =
 


=
Instruções para entrar na lista, sair da lista e usar a lista em
http://www.mat.puc-rio.br/~nicolau/olimp/obm-l.html
=


Re: [obm-l] Como resolve?

2005-04-07 Por tôpico Claudio Buffara
on 07.04.05 19:43, Bruno Bruno at [EMAIL PROTECTED] wrote:

 x^3 + x^2 + x = 1000
 
 Como se faz? E como se resolve equações do tipo ax^3 + bx^2 + cx + d =
 0 onde b0 ?
 
Faz x = y + m, e acha o valor de m tal que a equacao em y nao tenha termo em
y^2. Dai usa a formula.
 


=
Instruções para entrar na lista, sair da lista e usar a lista em
http://www.mat.puc-rio.br/~nicolau/olimp/obm-l.html
=


Re: [obm-l] limsup e subsequencias

2005-04-07 Por tôpico Claudio Buffara
on 07.04.05 22:22, Fabio Niski at [EMAIL PROTECTED] wrote:

 Ola pessoal.
 Me deparei com o seguinte problema:
 Seja X = (x[n]) uma sequencia limitada em R.
 Prove que se L é o conjunto dos v pert R tal que exista uma subsequencia
 de X que converge para v, entao limsup(x[n]) = sup L
 
 Bom o que eu consegui até agora foi isso:
 Suponha que exista uma subsequencia que convirja para um numero v maior
 do que limsup(x[n]).
 Ora, como v é limite de uma subsequencia de (x[n]) entao existem
 infinitos termos da sequencia que estao no intervalo (v-eps, v+eps) para
 qualquer eps  0. Em particular existem infinitos indices n tal que
 x[n]  limsup(x[n]), mas isto é uma contradicao pois limsup(x[n]) é
 justamente o menor elemento de (x[n]) tal que existam apenas um numero
 finito de elementos de (x[n]) maior do que ele.
 
 Acredito que eu mostrei aqui que limsup(x[n]) é apenas um limitante
 superior para L certo? Como eu mostro que ele é o menor limitante
 superior (e portanto o sup) de L ?

Seja a = limsup(x(n)).

Se a nao for limite de uma subsequencia de (x(n)) entao existe eps  0 tal
que o intervalo (a - eps,a + eps) nao contem termo algum de (x(n)).

Isso quer dizer que existe N tal que n = N == x(n) = a - eps, pois voce
jah provou acima que existe no maximo uma quantidade finita de indices n
tais que x(n) = a + eps.

Logo, a(N) = sup{x(n) | n = N} = a - eps. Em particular, a(N) nao pode
convergir para a == contradicao, pois lim(N - inf) a(N) = a (lembre-se da
definicao de limsup)

Portanto, existe uma subsequencia de (x(n)) convergindo para a.

Como nenhuma subsequencia pode convergir para algum real maior do que a,
concluimos que a = sup(L).


[]s,
Claudio.


=
Instruções para entrar na lista, sair da lista e usar a lista em
http://www.mat.puc-rio.br/~nicolau/olimp/obm-l.html
=


Re: [obm-l] limsup e subsequencias

2005-04-07 Por tôpico Claudio Buffara
on 07.04.05 22:22, Fabio Niski at [EMAIL PROTECTED] wrote:

...
 Em particular existem infinitos indices n tal que x[n]  limsup(x[n]),
 mas isto é uma contradicao pois limsup(x[n]) é
 justamente o menor elemento de (x[n]) tal que existam apenas um numero
 finito de elementos de (x[n]) maior do que ele.
...

A afirmacao correta eh:
Se b  limsup(x(n)) entao existe apenas uma quantidade finita de indices n
tais que x(n) = b. Alem disso, limsup(x(n)) eh o infimo do conjunto B de
tais b, mas nao pertence necessariamente a este conjunto. Ou seja, pode
haver uma infinidade de indices n tais que x(n)  limsup(x(n)).
Por exemplo: x(n) = 1/n  0 para todo n, e limsup(x(n)) = lim x(n) = 0.
Por outro lado, se y(n) = 1 - 1/n, entao limsup(y(n)) = lim y(n) = 1 mas
nenhum termo de y(n) eh maior que ou igual a 1.

Pra provar a primeira afirmacao, suponha que b  a = limsup(x(n)) e que haja
uma infinidade de indices n tais que x(n) = b.
Seja a(n) = inf{x(m) | m = n}. (a(n)) eh monotona nao-crescente e limitada,
pois (x(n)) eh limitada. Pela definicao de limsup, a = lim a(n).
Para todo n, existe n_1 tal que n_1  n e x(n_1) = b.
Logo, x(n_1) pertence a {x(m) | m = n} e, portanto,
a(n) = sup{x(m) | m = n} = x(n_1) = b.
Logo, lim a(n) = b  a ==
contradicao == 
existe apenas um numero finito de indices n tais que x(n) = b.

Seja agora c  a.
Nos provamos, no e-mail anterior, que (x(n)) tem uma subsequencia
convergindo pra a. Assim, dado eps  0, o intervalo (a - eps,a + eps) contem
uma infinidade de termos da sequencia. Em particular, tomando eps = a - c,
vemos que existe uma infinidade de termos maiores do que c.
Logo, se c  a, entao c nao pertence a B.

Conclusao inescapavel: a = inf(B).
Mais ainda: B contem (a,+inf) e estah contido em [a,+inf).

***

Valeu, Niski. Essa sua duvida me forcou a relembrar a teoria de limsup e
liminf, que eu nao via desde o ano passado, e a escrever as demonstracoes,
coisa que eu nunca havia feito. Foi um bom treino. Obrigado.

[]s,
Claudio.


=
Instruções para entrar na lista, sair da lista e usar a lista em
http://www.mat.puc-rio.br/~nicolau/olimp/obm-l.html
=


Re: [obm-l] RE: [obm-l] Quadrado Mágico

2005-04-06 Por tôpico Claudio Buffara
on 06.04.05 22:14, [EMAIL PROTECTED] at [EMAIL PROTECTED] wrote:

 Oi, Cláudio
 
 ''-- Mensagem Original --
 ''Date: Wed,  6 Apr 2005 17:46:51 -0300
 ''Subject: [obm-l] Quadrado Mágico
 ''From: claudio.buffara [EMAIL PROTECTED]
 ''To: obm-l obm-l@mat.puc-rio.br
 ''Reply-To: obm-l@mat.puc-rio.br
 ''
 ''
 ''Acho que sei como demonstrar que L_i (1=i=n), C_j (1=j=n-1), T e
 S são
 ''funcionais lineares L.I.
 ''
 ''Suponhamos que existam escalres a_i (1=i=n), b_j (1=j=n), c e d
 tais
 ''que o funcional linear:
 ''F = SOMA(1...n) a_i*L_i + SOMA(1...n-1) b_j*C_j + c*T + d*S
 ''seja identicamente nulo.
 ''
 ''Seja A(i,j) a matriz cujo coeficiente (i,j) é 1 e todos os demais são
 0.
 ''
 ''F(A(1,n)) = 0 == a_1 + d = 0
 ''F(A(n,n)) = 0 == a_n + c = 0
 ''F(A(k,n)) = 0 para 2 = k = n-1 ==  a_k = 0.
 ''
 ''Ou seja, já podemos escrever:
 ''F = -d*L_1 - c*L_n + SOMA(1..n-1) b_j*C_j + c*T + d*S.
 ''
 ''F(A(1,1)) = 0 == -d + b_1 + c = 0
 ''F(A(2,1)) = 0 == b_1 = 0 == c = d
 ''F(A(k+1,k)) = 0 para 2 = k = n-2 == b_k = 0
 ''F(A(n,n-1)) = 0 == -c + b_(n-1) = 0
 ''
 ''Assim:
 ''F = c*(-L_1 - L_n + C_(n-1) + T + S).
 ''
 ''Finalmente, F(A(2,2)) = 0 == c = 0.
 ''
 ''Logo, a_i = b_j = c = d = 0 e, portanto, os 2n+1 funcionais acima são
 L.I.
 ''e, portanto, o espaço dos quadrados mágicos nxn tem dimensão n^2 - (2n+1)
 ''= n^2 - 2n - 1.
 ''
 ''[]s,
 ''Claudio.
 
 A demonstração da independência dos funcionais está ok, mas isso mostra
 que se Z é o conjunto das matrizes n x n tais que todos esses funcionais
 se anulam, então Z (na verdade um subespaço de M(nxn)) é tal que dim Z =
 dim M(nxn) - dim(F), onde por F é o subespaço gerado pelos 2n + 2 funcionais
 em questão no dual de M(nxn), isto é, dim Z = n^2 - (2n + 1) = n^2 - 2n
 - 1. Este resultado é mais geral: Se Z é o espaço dos zeros dos funcionais
 f_1, ..., f_k, que estão no dual de V, então dim Z = dim V - dim (f_1, ...,
 f_k).
 
 Mas claramente a matriz A com todas as coordenadas 1 satisfaz L_1(A) = ...
 = L_n(A) = C_1(A) = ... = C_n(A) = T(A) = S(A) = n  0, assim dim Q =
 dim Z + 1 (Q = conjunto das matrizes A que satisfazem L_1(A) = ... = L_n(A)
 = C_1(A) = ... = C_n(A) = T(A) = S(A), ou seja, dos quadrados mágicos, um
 subespaço vetorial). Dá para provar que dim Q = dim Z + 1 (naturalmente
 um resultado mais geral válido para quaisquer funcionais), logo dim Q =
 dim Z + 1 = n^2 - 2n.
 
 []s,
 Daniel
 
Claro!

Eu esqueci justamente de levar em conta a condicao de quadrado magico...

Uma forma mais elementar de ver isso eh observar que o espaco dos quadrados
magicos eh justamente o espaco-solucao do seguinte sistema de 2n equacoes
lineares homogeneas (e linearmente independentes, como demonstrado acima) em
n^2 incognitas:
L_1 - T = 0
L_2 - T = 0
...
L_n - T = 0
C_1 - T = 0
C_2 - T = 0
...
C_(n-1) - T = 0
S - T = 0
Logo, o espaco solucao do sistema (igual a Q) tem dimensao n^2 - 2n.

[]s,
Claudio.


=
Instruções para entrar na lista, sair da lista e usar a lista em
http://www.mat.puc-rio.br/~nicolau/olimp/obm-l.html
=


Re: [obm-l] soma de termos

2005-04-06 Por tôpico Claudio Buffara
Title: Re: [obm-l] soma de termos



Oi, Marcio:

O que eu tinha em mente, quando falei em solucao algebrica, era abrir os numeros binomais e tentar simplificar o emaranhado de fatoriais resultante.
Mas como nao fui totalmente explicito, tenho que aceitar esta solucao indutiva. Talvez seja a vinganca da inducao pelo que eu andei falando a respeito dela :-)

[]s,
Claudio.

on 06.04.05 18:29, Marcio Cohen at [EMAIL PROTECTED] wrote:

 Oi Cláudio.. Realmente é muito mais legal uma demonstração combinatória: Considere o conjunto dos números 0,1,2,3,...,n. Você quer escolher um sequencia a1  a2  ...  a(2m+1) de 2m+1 elementos, o que pode ser feito de lado direito modos. Por outro lado, para cada k=0...n, voce pode escolher o elemento k como sendo o termo do meio dessa sequencia, e então precisa escolher binomial(k,m) termos menores e binomial(n-k,m) termos maiores que k. Somando em k, vemos que a resposta é o lado esquerdo e está provado.
 
 Mas não é tão feio fazer algebricamente..Vamos generalizar e provar que Soma(k=0..n) Binomial(k,a)*Binomial(n-k,b) = Binomial (n+1,a+b+1)
 
 Por inducao em n. Para n=0 eh facil. Supondo valido para n fixo e a,b quaisquer, temos:
Soma(k=0..n+1) Binomial(k,a)*binomial (n+1-k,b) = Soma(k=0..n) Binomial(k,a)*[Binomial(n-k,b)+Binomial(n-k,b-1)] + Binom(n+1,a)*Binom(0,b)
Usando a hipotese indutiva, isso da: Binomial(n+1,a+b+1) + Binomial(n+1, a+b) = Binomial (n+2, a+b+1)
 Em particular, fazendo a=b=m voce tem a solucao do problema pedido ;) (tá, confesso que tentei fazer a indução direto antes e não consegui :) E demorei bem menos pra dar a solução combinatória do que por indução.. mas não resisti ao quero ver alguém ... :) 


 Abraços,
 Marcio
 
- Original Message - 
From: claudio.buffara mailto:[EMAIL PROTECTED] 
To: obm-l mailto:obm-l@mat.puc-rio.br 
Sent: Wednesday, April 06, 2005 3:58 PM
Subject: Re: [obm-l] soma de termos

Por exemplo, é possível dar uma demonstração combinatória da identidade abaixo, que foi uma questão da famosa e difícil prova do IME de 1980/81.
 
SOMA(k=0...n) Binom(k,m)*Binom(n-k,m) = Binom(n+1,2m+1).
 
Agora, quero ver alguém provar isso algebricamente...
 







Re: [obm-l] [x^n] == n (mod 2)

2005-04-06 Por tôpico Claudio Buffara
Title: Re: [obm-l] [x^n] == n (mod 2)



Me enganei (mais uma vez...)

O problema abaixo eh valido, mas eh trivial (eu me dei conta disso no caminho pra casa).

Mais interessante eh o seguinte: ache x real tal que [x^n] tem paridade oposta a de n.

E o que o Shine exibiu foi um numero NAO-INTEIRO x tal que [x^n] eh sempre impar.
(se x puder ser inteiro, basta tomar x = 1 ou x = impar qualquer).

Tambem eh trivial exibir um nao-inteiro x tal que [x^n] eh sempre par.

Alias, eh curioso que [x^n] sempre par ou [x^n] de mesma paridade que n sao problemas bem mais faceis do que [x^n] sempre impar ou [x^n] de paridade oposta a de n.

No mais, prove ou de um contra-exemplo:
1) dado um racional positivo qualquer x, sempre vai existir um inteiro positivo n tal que:
[x^n] e [x^(n+1)] tem a mesma paridade.

2) existe um numero real x tal que [x^n] eh primo para todo inteiro positivo n.

[]s,
Claudio.
 
on 06.04.05 17:01, claudio.buffara at [EMAIL PROTECTED] wrote:

Aqui vai um bonitinho:
 
Ache um número real x tal que, para todo n inteiro e positivo, [x^n] tem a mesma paridade que n.
 
[a] = maior inteiro que é menor ou igual a a.
 
Se não me engano, há algum tempo, o Shine exibiu um y tal que [y^n] é sempre ímpar.
 
[]s,
Claudio.
 
 







Re: [obm-l] [x^n] == n (mod 2)

2005-04-06 Por tôpico Claudio Buffara
on 06.04.05 23:13, Domingos Jr. at [EMAIL PROTECTED] wrote:

 claudio.buffara wrote:
 
 Aqui vai um bonitinho:
 
 Ache um número real x tal que, para todo n inteiro e positivo, [x^n]
 tem a mesma paridade que n.
 
 [a] = maior inteiro que é menor ou igual a a.
 
 Se não me engano, há algum tempo, o Shine exibiu um y tal que [y^n] é
 sempre ímpar.
 
 []s,
 Claudio.
 
 
 
 Você quer um x irracional, Cláudio?
 
 [ ]'s


Me mostre tres valores de x: um racional, um irracional algebrico e um
transcendente.



=
Instruções para entrar na lista, sair da lista e usar a lista em
http://www.mat.puc-rio.br/~nicolau/olimp/obm-l.html
=


Re: [obm-l] RE: [obm-l] Quadrado Mágico

2005-04-06 Por tôpico Claudio Buffara
on 06.04.05 22:14, [EMAIL PROTECTED] at [EMAIL PROTECTED] wrote:


 A demonstração da independência dos funcionais está ok, mas isso mostra
 que se Z é o conjunto das matrizes n x n tais que todos esses funcionais
 se anulam, então Z (na verdade um subespaço de M(nxn)) é tal que dim Z =
 dim M(nxn) - dim(F), onde por F é o subespaço gerado pelos 2n + 2 funcionais
 em questão no dual de M(nxn), isto é, dim Z = n^2 - (2n + 1) = n^2 - 2n
 - 1. Este resultado é mais geral: Se Z é o espaço dos zeros dos funcionais
 f_1, ..., f_k, que estão no dual de V, então dim Z = dim V - dim (f_1, ...,
 f_k).
 
 Mas claramente a matriz A com todas as coordenadas 1 satisfaz L_1(A) = ...
 = L_n(A) = C_1(A) = ... = C_n(A) = T(A) = S(A) = n  0, assim dim Q =
 dim Z + 1 (Q = conjunto das matrizes A que satisfazem L_1(A) = ... = L_n(A)
 = C_1(A) = ... = C_n(A) = T(A) = S(A), ou seja, dos quadrados mágicos, um
 subespaço vetorial). Dá para provar que dim Q = dim Z + 1 (naturalmente
 um resultado mais geral válido para quaisquer funcionais), logo dim Q =
 dim Z + 1 = n^2 - 2n.
 
 []s,
 Daniel
 
Claro!

Eu esqueci da condicao de que as matrizes sao quadrados magicos...

Uma forma mais elementar de ver isso eh observar que o espaco dos quadrados
magicos eh justamente o espaco-solucao de um sistema linear homogeneo de 2n
equacoes em n^2 incognitas. As equacoes sao:
L_1 - T = 0
...
L_n - T = 0
C_1 - T = 0
...
C_(n-1) - T = 0
S - T = 0
Como jah vimos, estas equacoes sao L.I. jah que os funcionais lineares
correspondentes sao L.I.
Logo, a dimensao do espaco solucao eh n^2 - 2n = dim(Q).

[]s,
Claudio.
 


=
Instruções para entrar na lista, sair da lista e usar a lista em
http://www.mat.puc-rio.br/~nicolau/olimp/obm-l.html
=


Re: [obm-l] Cálculo de Probabilidades e Teoria da Medida

2005-03-31 Por tôpico Claudio Buffara
on 30.03.05 08:08, Henrique Patrício Sant'Anna Branco at [EMAIL PROTECTED]
wrote:

 Pessoal,
 
 Estou estudando Cálculo de Probabilidades de uma visão um pouco mais
 avançada, ao ponto de despertar minha curiosidade sobre a Teoria da Medida.
 
 Alguém pode me indicar bons livros/sites para pesquisa, bem como os
 pré-requisitos pra estudar o assunto?
 
 Grato,
 Henrique.
 
A cadeira de Medida e Integracao no IMPA tem esta bibliografia:
BARTLE, R. - The Elementos of Integration, New York, J. Wiley, 1966.
FERNANDEZ, P. - Medida e Integração. Rio de Janeiro, IMPA, Projeto Euclides,
1976. 
ROYDEN, M. - Real Analysis. New York, The MacMillan (1963).
RUDIN, W. - Real and Complex Analysis. New York, Mc-Graw Hill, 1966.

Os especialistas falam muito bem do livro do Bartle, mas eu nao o conheco.

Alem do livro do P. Fernandez, o projeto Euclides lancou recentemente um
outro livro sobre o assunto, cujo autor agora me escapa.

De vez em quando, eu dou uma lida em partes do Lebesgue Integration and
Measure de Alan J. Weir - Cambridge University Press. Este eh o volume 1 de
uma publicacao em 2 volumes, eh mais elementar do que os dois do projeto
Euclides, e tem uma vantagem: vem com solucoes para os exercicios propostos.
Quem estuda sozinho sabe o quanto isso ajuda...

Pre-requisitos: O mais importante eh ter uma boa base de analise real.
Tambem ajuda se voce manjar de topologia geral e de teoria dos conjuntos.

Sobre sites, basta entrar no Google e escrever measure theory notes ou
measure pdf notes ou algo do genero que voce vai achar um monte de
referencias. Eu jah achei muita coisa interessante desse jeito, sobre
diversos assuntos, mas nem tudo eh de boa qualidade.

[]s,
Claudio.


=
Instruções para entrar na lista, sair da lista e usar a lista em
http://www.mat.puc-rio.br/~nicolau/olimp/obm-l.html
=


Re: [obm-l] questão de olimpíada

2005-03-31 Por tôpico Claudio Buffara
on 31.03.05 12:16, Felipe Nardes at [EMAIL PROTECTED] wrote:

 Ae galera me dá uma ajuda nessa questão:
 
 Determine todas as sequências finitas de números naturais consecutivos cuja
 soma seja igual a 1000.
 
 gabarito: (1000), (198,199,200,201,202), (55,56,57,...,69,70) e
 (28,29,30,...,51,52)
 
 valeu!
 
Se a sequencia tem um numero impar 2m+1 de termos entao 2m+1 divide 1000,
pois a soma de 2m+1 inteiros consecutivos eh igual a 2m+1 vezes o inteiro do
meio, igual a 1000/(2m+1).

Os divisores impares de 1000 = 2^3*5^3 sao 1, 5, 25 e 125.
Os termos do meio respectivos sao 1000, 200, 40 e 8.
Repare que 125 nao serve pois a sequencia correspondente teria termos
negativos, contrariamente ao enunciado.
Logo, teremos 3 sequencias com um numero impar de termos:
1 termo == (1000)
5 termos == (198,199,200,201,202)
25 termos (28,29,...,40,...,51,52)

Se a sequencia tem um numero par 2m de termos, entao vai existir um inteiro
positivo N tal que 2m*(N + 1/2) = 1000 == m*(2N + 1) = 1000.
A sequencia serah: (N-m+1, N-m+2, ..., N-1, N, N+1,...,N+m-1, N+m)
Obviamente, N-m+1 = 1 == N = m.
 
2N + 1 eh um divisor impar de 1000 e eh =3 ==
2N + 1 soh pode assumor os valores 5, 25 ou 125 ==
N soh pode ser 2, 12 ou 62 ==
os m correspendentes serao 200, 40 e 8 ==
Soh podemos ter N = 62  8 e a sequencia serah:
(55, 56, ..., 62, 63, 64, ..., 69, 70)

[]s,
Claudio.


=
Instruções para entrar na lista, sair da lista e usar a lista em
http://www.mat.puc-rio.br/~nicolau/olimp/obm-l.html
=


Re: [obm-l] ++Duvidas

2005-03-31 Por tôpico Claudio Buffara
Title: Re: [obm-l] ++Duvidas



Ache as solucoes de 10m + n = 3*m*n, onde m, n pertencem a {0,1,2,3,4,5,6,7,8,9} e m  0.

on 31.03.05 12:54, matduvidas48 at [EMAIL PROTECTED] wrote:

.Sejam A e B dois números de dois algarismos cada um e AB. 

Sabendo-se que cada um desses números é igual ao triplo do produto de seus algarismos, qual a razão A/B? 

 

 

 Fico agradecido 

 

 

Ary Queiroz







Re: [obm-l] Primo ou composto??? (correção)

2005-03-31 Por tôpico Claudio Buffara
Esse problema tah meio esquisito.

Por exemplo, se p+2 for composto (casos de p = 2, 7, 13, 19, 23, 31, ...), o
menor valor de n eh obviamente 1.

Jah se p = 3, 5 ou 11, o menor valor de n eh mesmo p.

Por outro lado, se p = 17, entao n = 2 pois 2*2^2 + 17 = 25 = 5^2.
Alias, isso eh verdade para todo primo p terminado em 7 e tal que p+2 eh
primo, uma vez que se p = 10k+7, entao 2*2^2 + p = 10k + 15 = 5*(2k+3).

De onde voce tirou este problema?


on 31.03.05 16:01, Rhilbert Rivera at [EMAIL PROTECTED] wrote:

 
 Desculpe Qwert Smith ( mas, mesmo assim obrigado) me enganei na hora de
 escrever. Na realidade o problema é:
  Determine o menor valor positivo de n tal que  2.n^2 + p, seja um
 número inteiro  composto, onde p é um número primo.
 
 Como eu queria dizer, para n=p temos uma solução. Mas, existe solução para n
 menor que p? É aí que eu me atrapalho.
 
 P.S. Foi você Qwert que me escreveu uma vez solicitando livros
 disponibilizados LEGALMENTE e gratuitamente na internet, além daqueles que
 eu coloquei na lista?
 
 _
 MSN Messenger: converse online com seus amigos .
 http://messenger.msn.com.br
 
 =
 Instruções para entrar na lista, sair da lista e usar a lista em
 http://www.mat.puc-rio.br/~nicolau/olimp/obm-l.html
 =
 


=
Instruções para entrar na lista, sair da lista e usar a lista em
http://www.mat.puc-rio.br/~nicolau/olimp/obm-l.html
=


Re: [obm-l] Questão interessante ( Dos pesos distintos)

2005-03-20 Por tôpico Claudio Buffara
Title: Re: [obm-l] Questão interessante ( Dos pesos distintos)



on 20.03.05 16:00, Robÿe9rio Alves at [EMAIL PROTECTED] wrote:

DAdos n ( n maior ou igual do que 2 ) objetos de pesos distintos, prove que é possivel determinar qual o mais pesado fazendo 2n - 3 pesagens em uma balança de pratos. É esse número mínimo de pesagens que permitem determinar o mais leve e o mais pesado ? 

Como faz?

Faz por inducao.

Pra n = 2 eh obvio: colocamos um objeto em cada prato e vemos imediatamente qual o mais pesado, com apenas 2*2 - 3 = 1 pesagem.

Suponhamos que isso seja verdade pra n objetos.

Se tivermos n+1 objetos, separamos um deles e, em 2n - 3 pesagens, descobrimos qual o mais pesado dentro os n restantes (pela hipotese de inducao).

Em seguida, com mais uma pesagem (analoga ao caso n= 2) comparamos este objeto com aquele que separamos inicialmente.
Total = 2n - 3 + 1 = 2n - 2 = 2(n+1) - 4  2(n+1) - 3 pesagens. 

***

Este nao eh o numero minimo de pesagens.
 
O numero minimo de pesagens eh igual a n - 1.

P_1: compara a_1 e a_2, obtendo max(a_1,a_2);
P_2: compara max(a_1,a_2) e a_3, obtendo max(a_1,a_2,a_3);
...
P_(n-1): compara max(a_1,a_2,...,a_(n-1)) e a_n, obtendo max(a_1,a_2,...,a_n).

Repare que isso soh prova que o numero minimo de pesagens eh = n-1 mas, por inducao, dah pra mostrar que este eh de fato o minimo.


[]s,
Claudio.







[obm-l] Tres Probleminhas

2005-03-19 Por tôpico Claudio Buffara
Pra quem nao tah fazendo nada neste fim de semana...

1. Expressar o numero 19 usando uma unica vez cada um dos numeros 1, 2 e 3 e
mais as operacoes matematicas usuais (+, -, *, /, raizes, fatoriais, etc.).
Nao vale usar ponto decimal nem a funcao maior inteiro. (essa eh pro Qwert!)

2. Quanto vale lim(x - 1-) (x - x^2 + x^4 - x^8 + x^16 - x^32 + ...) ?

3. Sabe-se que a probabilidade de dois inteiros tomados ao acaso serem
primos entre si eh igual a 6/Pi^2. Tomando 4 inteiros a, b, c, d ao acaso (e
de forma independente) calcule a probabilidade de que mdc(a,b) = mdc(c,d).

[]s,
Claudio.

=
Instruções para entrar na lista, sair da lista e usar a lista em
http://www.mat.puc-rio.br/~nicolau/olimp/obm-l.html
=


Re: [obm-l] desigualdade curiosa...

2005-03-19 Por tôpico Claudio Buffara
Title: Re: [obm-l] desigualdade curiosa...



Se a e b sao ambos maiores do que 1 ou ambos menores do que 1, entao eh claro que a igualdade da hipotese nao pode ocorrer. 

Se a = b = 1, entao a^2 + b^2 = 2.

Logo, podemos supor s.p.d.g. que 0  a  1  b.

A igualdade fornece:
b^1999*(b^2 - 1) = a^1999*(1 - a^2) ==
(b/a)^1999 = (1 - a^2)/(b^2 - 1)  1, pois b  1  a ==
1 - a^2  b^2 - 1 ==
2  a^2 + b^2.
 
[]s,
Claudio.

on 19.03.05 19:03, carlos gomes at [EMAIL PROTECTED] wrote:

Seguindo a sugestão do Cláudio, essa é para quem está procurando um bom problema
 
Se a e b são números reais positivos tais que a^2001+b^2001=a^1999+b^1999, mostre que a^2+b^2 é menor do que ou igual a 2.
 
C.Gomes.
 
 
 






Re: [obm-l] Exerciacute;cios

2005-03-19 Por tôpico Claudio Buffara
Title: Re: [obm-l] Exerciacute;cios



on 19.03.05 19:08, Daniela Yoshikawa at [EMAIL PROTECTED] wrote:

Provar:
 
1) a + b = 0 ; a != 0; b != 0 - a/b^2 + b/a^2 = 1/a + 1/b
 != (diferente)

Eu prefiro escrever diferente como , mas eh soh questao de gosto...

a/b^2 + b/a^2 - (1/a + 1/b) =
(a^3 + b^3 - (ab^2 + a^2b))/(a^2b^2) =
(a + b)((a^2 - ab + b^2) - ab)/(a^2b^2) =
(a+b)(a - b)^2/(a^2b^2) = 0 
 

2) (a=0 b=0 c=0) - a + b + c = ãab + ãbc + ãac

O lado direito da desigualdade nao ficou legivel no meu computador.
Procure nao usar caracteres especiais.

 
3) (a=0 b=0 c=0) - ab(a+b) + bc(b+c) + ac(a+c) = 6abc

Como os numeros sao nao-negativos, vale a desigualdade MG = MA.
Assim: 
(abc)^(1/3) = (a+b+c)/3
e
(ab*ac*bc)^(1/3) = (ab+ac+bc)/3.

Multiplicando estas duas desigualdades, obtemos:
abc = (a+b+c)(ab+ac+bc)/9 ==

(ab+ac+bc)(a+b+c) - 9abc = 0 ==

ab(a+b+c) - abc + bc(a+b+c) - abc + ac(a+b+c) - abc - 6abc = 0 ==

ab(a+b) + bc(a+b) + ac(a+b) - 6abc = 0 ==

ab(a+b) + bc(b+c) + ac(a+c) = 6abc.


[]s,
Claudio.





Re: [obm-l] Exerciacute;cios

2005-03-19 Por tôpico Claudio Buffara
Title: Re: [obm-l] Exerciacute;cios



on 19.03.05 19:08, Daniela Yoshikawa at [EMAIL PROTECTED] wrote:

Provar:
 
2) (a=0 b=0 c=0) - a + b + c = ãab + ãbc + ãac

Supondo que o que deve ser provado eh:
a + b + c = raiz(ab) + raiz(bc) + raiz(ac),

uma ideia eh somar as desigualdades:
a + b = 2raiz(ab)
a + c = 2raiz(ac)
b + c = 2raiz(bc)

Cada uma delas decorre de (raiz(x) - raiz(y))^2 = 0.


[]s,
Claudio.





Re: [obm-l] Indução

2005-03-18 Por tôpico Claudio Buffara
Oi, Marcio:

Da pra provar ainda mais: que (1 + 1/n)^n  3 para todo n.

Uma ideia legal eh expandir (1 + 1/n)^n usando o binomio de Newton, dar uma
arrumada na expressao resultante e deduzir que ela eh limitada superiormente
por: 
1 + 1/1! + 1/2! + 1/3! + ... + 1/n!,
a qual por sua vez eh limitada superiormente por:
1 + 1 + 1/(1*2) + 1/(2*3) + ... + 1/((n-1)*n) =
1 + 1 + (1 - 1/2) + (1/2 - 1/3) + ... + (1/(n-1) - 1/n) =
1 + 1 + (1 - 1/n)  3.

Dai usando a desigualdade MG  MA com os n+1 numeros:
1 + 1/n, 1 + 1/n, ..., 1 + 1/n, 1
(ou seja n numeros iguais a 1 + 1/n e 1 numero igual a 1)
voce obtem:
(1 + 1/n)^(n/(n+1))  1 + 1/(n+1) ==
(1 + 1/n)^n  (1 + 1/(n+1))^(n+1) ==
((1 + 1/n)^n) eh crescente.

Logo, ((1 + 1/n)^n) eh monotona crescente e limitada superiormente por 3.
Assim, existe lim(n - infinito) (1 + 1/n)^n.


[]s,
Claudio.

on 17.03.05 22:22, Marcio M Rocha at [EMAIL PROTECTED] wrote:

 Boa noite, pessoal.
 A questão abaixo também consta do Vol. 1 de A Matemática do Ensino Médio.
 Ela tem duas partes, das quais fiz a primeira. Gostaria de pedir que alguém
 verificasse se está tudo OK.
 
 Parte 1) Prove que ((n + 1)/n) elevado a n =n para todo n=3.
 
 Para n = 3 temos (4/3)³ =3
 
 Solução
 Supondo verdadeira para algum k3:
 
 ((k + 1)/k) elevado a k =k
 
 Multiplico a desigualdade acima por ((k + 1)/k) e obtenho
 
 ((k + 1)/k)elevado a (k + 1) = k + 1
 
 Só que quando k  3, (k + 2)/(k + 1) = (k + 1)/k, e daí:
 
 ((k + 2)/(k + 1)) elevado a (k + 1)  = ((k + 1)/k) elevado a (k + 1)
 
 Logo (((k + 1) + 1)/(k + 1)) elevado a (k + 1) = k + 1
 
 Parte 2) Use esse fato para mostrar que a seqüência
 
 1, 2¹/2, 3¹/3, 4¹/4, ...
 
 é decrescente a partir do 3o termo.
 
 Esta parte ainda está saindo.
 
 Desculpem se são questões triviais para vocês.
 
 Abraços.
 
 Márcio.
 
 =
 Instruções para entrar na lista, sair da lista e usar a lista em
 http://www.mat.puc-rio.br/~nicolau/olimp/obm-l.html
 =
 


=
Instruções para entrar na lista, sair da lista e usar a lista em
http://www.mat.puc-rio.br/~nicolau/olimp/obm-l.html
=


Re: [obm-l] Proposição

2005-03-18 Por tôpico Claudio Buffara
on 17.03.05 21:25, [EMAIL PROTECTED] at [EMAIL PROTECTED] wrote:

 Einstein falou uma frase que toca no que você escreveu:
 A inovação não é o produto de um pensamento lógico, mesmo estando o produto
 final atado a uma estrutura lógica.
 
Esta deve ser uma das razoes pelas quais dizem que o principio da inducao
matematica nao serve para descobrir teoremas, mas apenas para prova-los.
Isso talvez ocorra justaente porque ele eh um axioma atado a estrutura
logica que define e descreve os numeros naturais (e, por conseguinte, todos
os outros numeros).
Apesar disso, em alguns casos, a passagem de n para n+1 requer bastante
criatividade, ou seja, alguma inovacao que nao estah contida (pelo menos nao
explicitamente) no encadeamento logico da teoria dos numeros naturais.

 E sobre o teorema do fechamento algébrico dos complexos, o livro do
 Rudin Principles of mathematical analysis tem uma prova curtinha e não
 muito difícil, e os pré-requisitos para compreendê-la estão todos dentro do
 livro.

Eu continuo achando que pelo menos tao importante quanto conhecer a
demonstracao rigorosa de algum teorema, eh conhecer a intuicao por tras
dela. No caso desse ai, eu acho muito instrutivo ver o que acontece com a
imagem da circunferencia |z| = R por um dado polinomio em C[z] quando R
varia de 0 a um valor muito grande, de modo que a imagem varia de um ponto
no plano complexo a algo muito proximo da circunferencia |z| = R^n, onde n
eh o grau do polinomio. Em algum instante, esta imagem vai passar pela
origem e isso significa que o polinomio tem alguma raiz.

 Para aproveitar o espaço: Alguém sabe exibir uma base para o espaço vetorial
 das seqüências reais (R^oo)? Ou ainda, alguém conhece uma base para o espaço
 das seqüências formadas por 0 e 1?

Imagino que este ultimo possa ser visto como um espaco vetorial sobre Z_2.
Como o conjunto destas sequencias eh nao enumeravel, uma base desse espaco
tem que ser nao-enumeravel.

O primeiro espaco eh isomorfo ao espaco vetorial real das funcoes de N em R.
Se nao me engano, o conjunto de tais funcoes tem a mesma cardinalidade de R.
Assim, serah que ele nao possui base enumeravel? Eu nao tenho certeza.
 
Um outro exemplo de espaco com base nao enumeravel eh o espaco vetorial dos
reais sobre os racionais. Dah pra provar que embora ele proprio nao seja uma
base, o conjunto de Cantor contem uma tal base.

[]s,
Claudio.


=
Instruções para entrar na lista, sair da lista e usar a lista em
http://www.mat.puc-rio.br/~nicolau/olimp/obm-l.html
=


Re: [obm-l] Proposição

2005-03-18 Por tôpico Claudio Buffara
Eu tenho uma duvida:

Tenho quase certeza de que R^N tem a mesma cardinalidade de R.
Serah que, nesse caso, a base precisa mesmo ser nao-enumeravel?

[]s,
Claudio.

on 18.03.05 07:26, Bernardo Freitas Paulo da Costa at [EMAIL PROTECTED]
wrote:

 Depende do que você está pensando. Se for apenas uma base no sentido
 de Hamel, ou seja, todo elemento é escrito como uma ÚNICA combinação
 linear FINITA dos elementos da base, dá para provar que estas bases
 são não-enumeráveis. Assim, pode ser difícil exibir uma base. Por
 exemplo, no segundo, você pode pensar que uma seqüência é a
 representação binária de um número em [0, 1], mas ainda não sei se é
 bonitinho...
 
 Abraços,


=
Instruções para entrar na lista, sair da lista e usar a lista em
http://www.mat.puc-rio.br/~nicolau/olimp/obm-l.html
=


[obm-l] R^N ~ R

2005-03-18 Por tôpico Claudio Buffara
Lembrei da demonstracao de que R^N tem a mesma cardinalidade de R.

Sabemos que: 
R ~ 2^N (conjunto das funcoes de N em {0,1})
e 
N ~ NxN (conjunto dos pares ordenados de numeros naturais).

Logo, R^N ~ (2^N)^N ~ 2^(NxN) ~ 2^N ~ R.

Explicitamente, as bijecoes f: R - 2^N  e  g: N - NxN
induzem as bijecoes:
F: R^N - (2^N)^N  e  G: 2^N - 2^(NxN),
dadas por:
F(a_1, a_2, a_3, ...) = (f(a_1), f(a_2), f(a_3), ...)
e
G(n_1, n_2, n_3, ...) = (g(n_1), g(n_2), g(n_3), ...)

F eh injetiva, pois:
F(a_1, a_2, ...) = F(b_1, b_2, ...) ==
f(a_i) = f(b_i) para i = 1, 2, ... ==
a_i = b_i para i = 1, 2, ... (jah que f eh injetiva) ==
(a_1, a_2, ...) = (b_1, b_2, ...)

F eh sobrejetiva, pois:
dado (b_1, b_2, ...) em (2^N)^N, existem a_1, a_2, ... em R tais que b_1 =
f(a_1), b_2 = f(a_2), ... pois f eh sobrejetiva.
Logo, (b_1, b_2, ... ) = (f(a_1), f(a_2), ...) = F(a_1, a_2, ...).

Analogamente para G.

Agora definimos uma bijecao H: 2^(NxN) - (2^N)^N da seguinte forma:
Seja s: NxN - {0,1} uma funcao.
Seja (s_1, s_2, s_3, ...) uma sequencia cujos termos sao funcoes de N em
{0,1} definidas por s_i(j) = s(i,j)
Fazemos H(s) = (s_1, s_2, s_3, ...)

H eh injetiva pois:
H(s) = H(t) ==
(s_1, s_2, ...) = (t_1, t_2, ...) ==
s_i = t_i para i = 1, 2, ... ==
s_i(j) = t_i(j) para i = 1, 2, ... e j = 1, 2, ... ==
s(i,j) = t(i,j) para i = 1, 2, ... e j = 1, 2, ... ==
s = t

H eh sobrejetiva pois:
Dada (s_1, s_2, ...) em (2^N)^N, tomamos s em 2^(NxN) tal que:
s(i,j) = s_i(j) e, neste caso eh claro que H(s) = (s_1, s_2, ...).


Pra terminar eh soh reparar que a composta:
F^(-1) o H o G o f: R - R^N eh uma bijecao.
 

[]s,
Claudio.

=
Instruções para entrar na lista, sair da lista e usar a lista em
http://www.mat.puc-rio.br/~nicolau/olimp/obm-l.html
=


[obm-l] Elonzinho x Elonzao

2005-03-18 Por tôpico Claudio Buffara
Oi, Paulo:

Nao vejo nada de errado com o uso do Elonzinho (Analise Real - vol.1) ao
inves do Elonzao (Curso de Analise - vol.1), ateh porque este ultimo eh
razoavelmente enciclopedico e nao se pode esperar que um aluno normal de
graduacao o domine por completo. No mais, varios conceitos importantes
(sequencias de Cauchy, liminf e limsup me vem a cabeca) que nao estao
expostos no texto de livro menor constam dos exercicios propostos.

Na minha opiniao, e acho que jah escrevi bastante a esse respeito aqui na
lista, um problema muito mais grave do ensino da matematica nesse pais eh o
salto qualitativo que existe entre a matematica do ensino medio (calcule
isso) e a matematica universitaria (prove isso e/ou conjecture).

Esse problema eh tao serio que eu acho que todos os cursos de graduacao em
matematica deveriam oferecer uma cadeira que durasse todo o 1o. ano e que
poderia se chamar Introducao a Matematica Universitaria - 1 e 2.

O programa dessa cadeira seria equivalente, digamos, aos capitulos 1 a 5 do
Elonzinho, mais os capitulos 1 e 2 do Introduction to the Theory of
Numbers - Niven, Zuckerman e Montgomery, e as secoes 2.1 a 2.7 e 3.1 a 3.10
do Topics in Algebra - Herstein.

Um tal programa permitiria que os alunos se familiarizassem com conceitos
basicos de algebra e analise e com a arte de se fazer conjecturas e
demonstrar teoremas. Dessa forma, ao iniciar os cursos de algebra e analise
propriamente ditos a partir do 2o. ano, eles nao teriam o choque de se
deparar pela primeira vez com conceitos abstratos envolvendo epsilons,
deltas e grupos e aneis-quociente e com a necessidade de fazer demonstracoes
usando estes conceitos. Pelo contrario, dada a base com que os alunos
chegariam ao 2o. ano, estes cursos poderiam ir bem mais a fundo nos assuntos
do que vao hoje em dia, inclusive, no caso de analise na reta, cobrindo todo
o conteudo do Elonzao, como voce gostaria que fosse.

Obviamente, nao sou professor nem educador e nem mesmo formado em
matematica, de modo que minha opiniao nao deve valer quase nada. Enfim, aqui
estah...

***

Dito isso, dou todo o apoio a ideia de que comecemos a resolver aqui na
lista os problemas propostos no Elonzao.


[]s,
Claudio.

on 18.03.05 09:21, Paulo Santa Rita at [EMAIL PROTECTED] wrote:

 Agora, mudando de assunto, eu confesso que estou preocupado com um movimento
 que identifiquei  e que se relaciona com o excelente livro do Prof Elon L
 Lima, Curso de Analise, Vol 1, Projeto Euclides.
 
 Este livro indubitavelmente e uma bandeira contra a Mediocridade na
 Matematica, diferenciando-se para melhor em relacao a mesmice de uma imensa
 maioria de outros e, inexplicavelmente, vem sofrendo como que um boicote,
 nao sendo adotado como padrao e sendo substituido por um outro, do mesmo
 autor, que nao se diferencia em nada da maioria.
 
 Isso implica que se o estudante, por conta propria, nao se dedicar a estudar
 por ele, dificilmente tera oportunidade de em outros cursos ver analise na
 reta, carregando consigo portanto uma formacao mal feita com danosas
 consequencias na sua formacao e na formacao de Matematicos Brasileiros.
 
 Assim, salvo melhor juizo, para o bem da Matematica Brasileira, penso que
 todos os Institutos de Matematica Serios deveriam adota-lo como padrao,
 mesmo que fosse necessario dar 2 semestres para exaurir o seu conteudo num
 curso de graduacao.
 
 Aqui nesta lista nos podemos iniciar um movimento de reacao a esta tendencia
 mediocratizante, seja re-demonstrando os teoremas de forma mais clara, seja
 resolvendo os problemas mais dificeis. Assim, retirariamos um eventual
 receio que porventura seja provocado por isso. Em minha opiniao, este e um
 livro de formacao, nao e um livro de problema olimpicos. Quero dizer que (
 os exercicios ) eles nao sao sem graca como os triviais, mas nao chegam a
 ser desafiadores como os Olimpicos.
 
 Nos nao podemos permitir que este tesouro seja enterrado e esquecido.
 
 E com os melhores votos
 de paz profunda, sou
 
 Paulo Santa Rita
 6,0921,180305
 

=
Instruções para entrar na lista, sair da lista e usar a lista em
http://www.mat.puc-rio.br/~nicolau/olimp/obm-l.html
=


Re: [obm-l] Serie condicionalmente convergente

2005-03-18 Por tôpico Claudio Buffara
Falei besteira na minha msg anterior.

As bijecoes que sao produtos de ciclos finitos mantem a serie convergente e,
mais ainda, com a mesma soma, mas nao sao as unicas bijecoes que mantem a
convergencia, como o seu exemplo abaixo mostra.

No caso, a bijecao eh:
1 - 1
2 - 3
3 - 2

4 - 5
5 - 7
6 - 4

7 - 9
8 - 11
9 - 6

10 - 13
11 - 15
12 - 8

Ou seja, para cada n em N teremos:
f(3n-2) = 4n-3
f(3n-1) = 4n-1
f(3n) = 2n

***

Se S_n = a_1 + a_2 + a_3 + ... + a_n for a n-esima reduzida de uma serie
condicionalmente convergente, a reordenacao a ser buscada eh tal que a nova
reduzida passa ser:
R_n = S_n + T_n, onde T_n eh uma sequencia convergente

No seu exemplo:
R_2 = S_2 + (1/3)
R_4 = S_4 + (1/5 + 1/7)
R_6 = S_6 + (1/7 + 1/9 + 1/11)
R_8 = S_8 + (1/9 + 1/11 + 1/13 + 1/15)
...

Ou seja, T_n = 1/(n+1) + 1/(n+3) + ... + 1/(2n-1)  log(2), de modo que T_n
converge.

***

Vou ter que pensar mais um pouco no caso geral.

[]s,
Claudio.

on 18.03.05 09:21, Paulo Santa Rita at [EMAIL PROTECTED] wrote:

 Ola Claudio e demais colegas
 desta lista ... OBM-L,
 
 Voce ja o resolveu, apenas ainda nao percebeu isso ...  quando ha pouco voce
 exibiu A FUNCAO que so admite como conjuntos estaveis o VAZIO e o proprio X
 : basta generalizar esta funcao e aplica-la ao caso infinito, vale dizer, as
 re-ordenacoes dos indices da serie.
 
 A titulo de exemplificacao, considere o caso particular da serie
 condicionalmente convergente 1 - 1/2 + 1/3 - 1/4 + ... e a FUNCAO ( que voce
 ja percebeu ) que a reordena com o seguinte aspecto 1 + 1/3 - 1/2 + 1/5 +
 1/7 - 1/4 + 1/9 + 1/11 - 1/6 + ... . Claramente que as somas parciais podem
 ser colocadas assim :
 
 1 - 1/2 + (1/3)
 1- 1/2 + 1/3 - 1/4 + (1/5 + 1/7)
 
 A parte fora do parenteses e a soma antiga e a que esta dentro do parenteses
 e claramente convergente. Eu afirmo ( e neste caso particular e facil ver
 isso ) que em toda generalizacao da funcao a reordenacao resultante sera
 convergente. No caso geral, toma este caso particular como um limitante.
 
 Se nao me falha a memoria, eu dei uma sugestao que explica o restante.
 

=
Instruções para entrar na lista, sair da lista e usar a lista em
http://www.mat.puc-rio.br/~nicolau/olimp/obm-l.html
=


Re: [obm-l] ideais maximais

2005-03-18 Por tôpico Claudio Buffara
Faltou um detalhe trivial mas importante: provar que J eh de fato um ideal.

***

A reciproca eh mais legal.

Seja M um ideal maximal de C([0,1]).

Se f pertence a M entao, para algum x em [0,1], devemos ter f(x) = 0.
Caso contrario, 1/f pertenceria a C([0,1]) e, portanto, 1 = (1/f)*f
pertenceria a M, forcando M a ser igual a C([0,1]) == contradicao.

Agora, suponhamos que, para cada a em [0,1], existe alguma funcao f_a em M
tal que f_a(a)  0. Seja g_a = (f_a)^2 (ou seja, g_a(x) = (f_a(x))^2 para
todo x em [0,1]). 

Eh claro que g pertence a M e g_a(a)  0.
Como g_a eh continua, existe eps_a  0 tal que, para todo x no intervalo
aberto I_a de centro a e raio eps_a, g_a(x)  0.

Tambem eh claro que [0,1] eh coberto por Uniao(a em [0,1]) I_a.
Como [0,1] eh compacto, esta cobertura aberta admite uma subcobertura finita
I_a1 uniao I_a2 uniao ... uniao I_an.
Sejam g_a1, g_a2, ..., g_an as funcoes g correspondentes aos I_ai.
Entao h = g_a1 + g_a2 + ... + g_an pertence a M e eh tal que h(x)  0 para
todo x em [0,1] == contradicao.

Logo, existe z em [0,1] tal que, para toda f em M, f(z) = 0.

***

Se M eh um ideal maximal, entao o anel quociente C([0,1])/M eh um corpo.
Que corpo eh esse?

***

A demonstracao acima fura se o anel for C((0,1)), pois (0,1) nao eh
compacto. Quais sao os ideais maximais de C((0,1))?

[]s,
Claudio.

on 18.03.05 19:03, [EMAIL PROTECTED] at [EMAIL PROTECTED] wrote:

 Lista OBM ([EMAIL PROTECTED]) escreveu:
 
 Seja C([0,1]) o anel da funções contínuas em [0,1],
 com as operações (f + g)(x) = f(x) + g(x) e [f.g](x) =
 f(x).g(x), para todas f,g em C([0,1]). Seja J o
 conjunto de todas as funções f em C([0,1]) tais que
 f(1/2) = 0. Prove que J é um ideal maximal.
 
 Tome um ideal I contendo J, I diferente de J, isto é, existe h em I tal que h
 (1/2)  0. Então f(x) = h(x) - h(1/2) está em I, logo f(x) - h(x) = h(1/2)
  0 está em J. Como h(1/2) é escalar não nulo, segue que 1 está em J, logo
 J = C([0,1]).
 
 Vale também a recíproca: No anel C([0,1]), um ideal M é maximal se e somente
 se M é o conjunto das funções que se anulam num certo z, 0 = z = 1.
 
 []s,
 Daniel
 


=
Instruções para entrar na lista, sair da lista e usar a lista em
http://www.mat.puc-rio.br/~nicolau/olimp/obm-l.html
=


[obm-l] Corpos x Fields

2005-03-17 Por tôpico Claudio Buffara
Outra questao de jargao:

Em portugues, falamos CORPO.
Em alemao eh KORPER e em frances eh CORPS.

Por que nos paises de lingua inglesa eles falam FIELD?

Serah que acharam esquisito falar no BODY OF COMPLEX NUMBERS?

De mais a mais, nesse assunto, prefiro ficar com os alemaes e franceses.
Por que? Cite um grande matematico americano do seculo 19 (tah bom, tiveram
uns ingleses mas nao dao nem pra saida contra Gauss, Riemann, Cauchy e
Poincare...)
 
[]s,
Claudio.

on 17.03.05 09:32, Nicolau C. Saldanha at [EMAIL PROTECTED] wrote:

oeficientes a_j forem reais ou complexos.

[...]
 
 Aliás, campo provavelmente é uma tradução não usual de field.
 O termo usual e correto no nosso idioma é *corpo*.
 
 []s, N.
 


=
Instruções para entrar na lista, sair da lista e usar a lista em
http://www.mat.puc-rio.br/~nicolau/olimp/obm-l.html
=


[obm-l] Infimo e Integrais

2005-03-17 Por tôpico Claudio Buffara
on 17.03.05 11:41, Paulo Santa Rita at [EMAIL PROTECTED] wrote:
 
 Seja f:R-R uma funcao e S um conjunto qualquer, nao vazio. Para cada x em R
 definimos
 f(x)=INFIMO{|s-x|, s variando em S}. Prove que f:R-R e continua
 
Um bom problema eh calcular INTEGRAL(0..1) f(x)dx quando S eh o conjunto de
Cantor. A integral existe pois f eh continua e, portanto, integravel.

Ou entao, INTEGRAL(0..+infinito) f(x)dx quando S = {0} uniao {a_n | n em N}
onde a_n = 1 + 1/2 + ... + 1/n = n-esima reduzida da serie harmonica.
Se eu nao errei nas contas, essa eh mais uma aparicao inesperada de Pi,
dessa vez num contexto onde talvez o numero e fosse mais provavel, dado
que a serie harmonica eh intimamente relacionada ao logaritmo natural.

Um lema util eh o seguinte:
Se a e b (a  b) pertencem a S mas S inter (a,b) = vazio, entao:
Integral(a..b) f(x)dx = (b - a)^2/4.

[]s,
Claudio.

=
Instruções para entrar na lista, sair da lista e usar a lista em
http://www.mat.puc-rio.br/~nicolau/olimp/obm-l.html
=


[obm-l] Bijecao entre [0,1] e (0,1)

2005-03-17 Por tôpico Claudio Buffara
on 17.03.05 09:13, Paulo Santa Rita at [EMAIL PROTECTED] wrote:

 Ola Marcio e demais
 colegas desta lista ... OBM-L,
 
 Algumas problemas sobre funcoes e cardinalidade de conjuntos sao muito
 bonitos ... Lendo este ai embaixo eu me lembrei de alguns outros, tambem
 faceis mas que tem solucoes engenhosas :
 
Um que me ocorreu agora eh bem interessante, apesar de manjado:

Estabeleca uma bijecao entre os intervalos [0,1] e (0,1).

Mesmo manjado, vale o velho teorema:
piada velha + plateia nova = piada nova.


[]s,
Claudio.

=
Instruções para entrar na lista, sair da lista e usar a lista em
http://www.mat.puc-rio.br/~nicolau/olimp/obm-l.html
=


[obm-l] Serie condicionalmente convergente

2005-03-17 Por tôpico Claudio Buffara
Oi, Paulo:

Voce poderia dar a solucao deste problema?

[]s,
Claudio.

on 01.03.05 13:48, Paulo Santa Rita at [EMAIL PROTECTED] wrote:
 
 Seja A1 + A2 + ... + An + ... uma serie condicionalmente convergente.
 Caracterize as bijecoes
 f:N-N tais que
 Af(1) + Af(2) + ... + Af(n) converge.
 
 Nota : Af(n) = Termo da serie A1 + A2 + ... + An + ... cujo indice e f(n)
 
 SUGESTAO : note que facilmente voce pode criar uma sequencia semelhante a do
 exercicio que voce acabou de resolver ( inversos dos termos de uma PA ) e
 que converge para log(N)/N, qualquer que seja N. Ora, a expressao log(N)/N e
 bem conhecida e esta relacionada com um famoso teorema da teoria dos numeros
 ...
 
 Um Abraco
 Paulo Santa Rita
 3,1343,010305
 

=
Instruções para entrar na lista, sair da lista e usar a lista em
http://www.mat.puc-rio.br/~nicolau/olimp/obm-l.html
=


Re: [obm-l] Teorema de Cantor

2005-03-16 Por tôpico Claudio Buffara
on 16.03.05 21:30, Marcio M Rocha at [EMAIL PROTECTED] wrote:

 Pra ver se finalmente eu aprendo alguma coisa, estou resolvendo os problemas
 do volume 1 do livro A Matemática do Ensino Médio, do
 Elon/PC/Wagner/Morgado. Ainda não consegui o seguinte (para quem tem o livro,
 é o exercício 20 do capítulo 1):
 
 Prove o Teorema de Cantor: se A é um conjunto e P(A) é o conjunto das partes
 de A, não existe uma função f : A--P(A) que seja sobrejetiva.
 
 Muito obrigado.
 
 Márcio
 
 
A ideia eh supor que existe alguma f sobrejetiva e obter uma contradicao
envolvendo o conjunto B = {x pertencentes a A | x nao pertence a f(x)}

Obviamente, B pertence a P(A).

Como f eh sobrejetiva, existe a em A tal que f(a) = B.

De duas, uma: ou a pertence a B ou a nao pertence a B.
O que acontece em cada caso?

[]s,
Claudio.




=
Instruções para entrar na lista, sair da lista e usar a lista em
http://www.mat.puc-rio.br/~nicolau/olimp/obm-l.html
=


Re: [obm-l] C-homogeneidade implica C-Linearidade?

2005-03-16 Por tôpico Claudio Buffara
Tem aquele exemplo famoso (?) de uma funcao F:R - R que satisfaz a F(x + y)
= F(x) + F(y) mas que eh descontinua em toda a reta.

A ideia eh tomar uma base {r_i} (necessariamente nao enumeravel) de R sobre
Q e, dado o real x = a_1*r_1 + ... + a_n*r_n (a_i: racionais; r_i: elementos
da base) definir F(x) = a_1 + ... + a_n.

Pela definicao e propriedades de uma base, temos F(x + y) = F(x) + F(y).

Alem disso, eh claro que F(x) eh racional para cada x real, de modo que, se
a eh irracional e x  0, entao F(ax)  aF(x).

[]s,
Claudio.

on 16.03.05 20:18, Sergio Lima Netto at [EMAIL PROTECTED] wrote:

 
 Pelo que entendi a funcao  era definida como:
 
 T([a1 a2]) = 
 i) [a1 a2], se a1!=a2; (eu prefiro a1  a2)
 ii) [0 0], se a1=a2;
 
 Assim e´ facil ver que ela e´ homogenea
 (testa cada um dos dois casos). E´ facil ainda ver
 que ela nao e´ linear pois, para a != 0:
 
 f([a 0]) = [a 0];
 f([0 a]) = [0 a];
 f([a 0] + [0 a]) = f([a a]) = [0 0];
 
 E com isto, f([a 0] + [0 a]) e´ diferente
 de f([a 0]) + f([0 a]).
 
 Se possivel, ggostaria de colocar uma outra pergunta:
 Sera´ que alguem cita uma funcao que satisfaz a propriedade
 f(x1 + x2) = f(x1) + f(x2),
 mas nao satisfaz a propriedade
 f(kx1) = k f(x1)
 
 Seria o inverso so exemplo anterior.
 
 Abracos,
 sergio
 
 On Wed, 16 Mar 2005, Bernardo Freitas Paulo da Costa wrote:
 
 Oi, Cláudio. Esta função é exatamente
 T(z) = z/2 = Re(z) != Im(z)
 T(a + a*i) = 0, para a = 0
 
 Ou seja, ela é quase T(z) = z/2.
 
 Certo?
 
 
 =
 Instruções para entrar na lista, sair da lista e usar a lista em
 http://www.mat.puc-rio.br/~nicolau/olimp/obm-l.html
 =
 


=
Instruções para entrar na lista, sair da lista e usar a lista em
http://www.mat.puc-rio.br/~nicolau/olimp/obm-l.html
=


Re: [obm-l] C-homogeneidade implica C-Linearidade?

2005-03-16 Por tôpico Claudio Buffara
on 16.03.05 17:43, Bernardo Freitas Paulo da Costa at [EMAIL PROTECTED]
wrote:

 Oi, Cláudio. Esta função é exatamente
 T(z) = z/2 = Re(z) != Im(z)
 T(a + a*i) = 0, para a = 0
 
 Ou seja, ela é quase T(z) = z/2.
 
 Certo?

Certo. Eu nao tinha percebido a definicao diferente de T(a + a*i).

[]s,
Claudio.


=
Instruções para entrar na lista, sair da lista e usar a lista em
http://www.mat.puc-rio.br/~nicolau/olimp/obm-l.html
=


Re: [obm-l] autovalor

2005-03-16 Por tôpico Claudio Buffara
Title: Re: [obm-l] autovalor



on 16.03.05 18:38, Lista OBM at [EMAIL PROTECTED] wrote:

gostaria de uma ajuda no problema abaixo:
 
Sabe-se que A em M_2(C) tem um autocalor nulo. O que se pode dizer sobre A^(-1)? E sobre o determinante de A? E sobre o posto de A?
 

Notação: M_2(C) = conj. da matrizes 2x2 com coeficientes complexos.
 
 
grato desde já, éder.
A tem autovalor nulo ==
Av = 0 para algum vetor nao nulo v ==
A eh singular ==
A^(-1) nao existe, det(A) = 0 e posto(A) = 1.


[]s,
Claudio.





Re: [obm-l] Equao

2005-03-11 Por tôpico Claudio Buffara
Title: Re: [obm-l] Equao



Use a formula das raizes de uma equacao do 4o. grau.
Mas antes acho que voce precisa se livrar do termo de 3o. grau.

[]s,
Claudio.

on 11.03.05 15:17, Davidson Lima at [EMAIL PROTECTED] wrote:


 Encontrei, ontem, em um site as soluções (aproximadas), sendo: 

x1=-1,64282-0,460774i
x2=-1,64282+0,460774i
x3=-0,406095
x4=1,69174 

 Mas como faço para encontrá-las? 

 Felicidades! 

 Davidson Estanislau 
 
--- Alan Pellejero [EMAIL PROTECTED] wrote:






Re: [obm-l] Equao

2005-03-10 Por tôpico Claudio Buffara
Title: Re: [obm-l] Equao



Polinomio irredutivel sobre um corpo: polinomio que nao pode ser expresso como o produto de dois polinomios nao constantes com coeficientes nesse corpo.
Por exemplo, x^2 + 5x + 6 eh redutivel sobre Q pois eh igual a (x + 2)(x + 3).
Jah x^2 - 2 nao eh redutivel sobre Q, mas eh redutivel sobre R: (x + raiz(2))(x - raiz(2)).
Finalmente, x^2 + 1 nao eh redutivel sobre R mas eh sobre C: (x + i)(x - i).

Existe um teorema, devido a Gauss, que diz que se um polinomio com coeficientes inteiros eh redutivel sobre Q, entao ele eh redutivel sobre Z, ou seja, se f(x) pertence a Z[x] e f(x) = p(x)*q(x) com p(x) e q(x) em Q[x], entao existem polinomios p1(x) e q1(x) em Z[x] tais que f(x) = p1(x)*q1(x).

O criterio de Eisenstein diz o seguinte: dado f(x) = a_0 + a_1*x + ... + a_(n-1)*x^(n-1) + a_n*x^n em Z[x], 
se existe um primo p tal que: 
p divide a_0, a_1, ..., a_(n-1), 
p nao divide a_n, e
p^2 nao divide a_0,
entao f(x) eh irredutivel sobre Q.

As demonstracoes desses dois resultados nao sao muito dificeis mas sao um belo exercicio de teoria elementar dos numeros.

[]s,
Claudio.

on 10.03.05 08:07, Alan Pellejero at [EMAIL PROTECTED] wrote:

Caro amigo Cláudio...Se possível, gostaria de saber o que é um polinômio irredutível por Eisentein com p=2...Um grande abraço! 
 
x^4 + 2x^3 - 2x^2 - 6x - 2 = 0 ==
polinomio irredutivel por Eisenstein com p = 2 

Davidson Lima [EMAIL PROTECTED] wrote: 
 Não meu amigo, a equação é x^2+1/(x+1)^2=3. 

 Felicidades. 

 Davidson Estanislau

rt Smith [EMAIL PROTECTED] wrote:

From: Qwert Smith [EMAIL PROTECTED]
Date: Wed, 09 Mar 2005 23:08:06 -0500
To: obm-l@mat.puc-rio.br
Subject: Re: [obm-l] Equao

Sera que nao faltou um parentesis no numerador?

(x^2 + 1)/(x+1)^2=3


From: Claudio Buffara [EMAIL PROTECTED]

Fazendo o obvio ==
x^4 + 2x^3 + x^2 + 1 = 3x^2 + 6x + 3 ==
x^4 + 2x^3 - 2x^2 - 6x - 2 = 0 ==
polinomio irredutivel por Eisenstein com p = 2 ==
nao vejo nenhuma solucao bonitinha

Essa equacao tem uma unica raiz real, igual a aproximadamente 1,6917395.

[]s,
Claudio.

on 09.03.05 18:35, Davidson Lima at [EMAIL PROTECTED] wrote:


 Meus amigos, como fao para! resolver a questo:

 x^2+1/(x+1)^2=3

 Desde j agradeo a ateno.

 Davidson Estanislau



=
Instrues para entrar na lista, sair da lista e usar a lista em
http://www.mat.puc-rio.br/~nicolau/olimp/obm-l.html
=

 

The top resources for math --- http://www.Math.com/
Instruções para entrar na lista, sair da lista e usar a lista em http://www.mat.puc-rio.br/~nicolau/olimp/obm-l.html 

Yahoo! Acesso Grátis http://br.rd.yahoo.com/mail/taglines/*http://br.acesso.yahoo.com/ - Internet rápida e grátis. Instale o discador do Yahoo! agora. 






Re: [obm-l] 3 Problemas de Teoria dos N úmeros [EM INGLÊS]

2005-03-10 Por tôpico Claudio Buffara
on 10.03.05 18:41, Bruno Bruno at [EMAIL PROTECTED] wrote:

 What is the largest x for which 4^27 + 4^1000 + 4^x equals the square
 of a whole number?
 
 4^27 + 4^1000 + 4^x = n^2 = (a+b)^2 = a^2 + 2ab + b^2
 temos entao dois quadrados perfeitos, onde 4^x = 2ab e onde 4^x = b^2
 como queremos o maior x, 4^x = b^2
 
 a^2 + 2ab + b^2 = 4^27 + 4^1000 + 4^x = (2^27 + 2^x)^2
 
 2ab = 4^1000 = 2^2000 = 2*2^27*2^x = 2^(28+x)
 2000 = 28+x   - x = 1972

Acho que voce provou apenas que x = 1972.
O que impede x de ser maior do que 1972?

[]s,
Claudio.

=
Instruções para entrar na lista, sair da lista e usar a lista em
http://www.mat.puc-rio.br/~nicolau/olimp/obm-l.html
=


Re: [obm-l] ajuda(sequência)

2005-03-10 Por tôpico Claudio Buffara
Title: Re: [obm-l] ajuda(sequência)



on 10.03.05 14:16, cleber vieira at [EMAIL PROTECTED] wrote:

Esse eh interessante.

E acho que dah pra provar ainda mais: 
o limite eh igual a raiz(4*a_1 - 3), desde que a_1 = 3/4, 
apesar de eu nao ter ideia de como se faz isso.

O que acontece quando a_1  3/4?

[]s,
Claudio.

cleber vieira [EMAIL PROTECTED] wrote: 
Amigos, gostaria da ajuda de vocês neste problema que na verdade é dividido em três itens, entretanto, os outros dois já foram solucionados e este ainda não consegui resolver.Desde ja muito obrigado.
Os números a_1, a_2 , a_3,... são definidos como segue:
 
a_1 = 3/2 e
 a_(n+1) = [3(a_n)^2 + 4(a_n) - 3]/ 4(a_n)^2.
 
 
Determine lim (a_1)*(a_2)*(a_3)***(a_n)
com n tendendo a infinito.
Ass:Vieira

Yahoo! Acesso Grátis http://br.rd.yahoo.com/mail/taglines/*http://br.acesso.yahoo.com/ - Internet rápida e grátis. Instale o discador do Yahoo! agora.
__
Converse com seus amigos em tempo real com o Yahoo! Messenger 
http://br.download.yahoo.com/messenger/ 






Re: [obm-l] ajuda(sequência)

2005-03-10 Por tôpico Claudio Buffara
Title: Re: [obm-l] ajuda(sequência)



A recorrencia eh:
a_(n+1) = (3(a_n)^2 + 4a_n - 3)/(4(a_n)^2) ==
a_(n+1) = 3/4 + (4a_n - 3)/(4(a_n)^2) ==
(4a_(n+1) - 3)/4 = (4a_n - 3)/(4(a_n)^2) ==
4a_(n+1) - 3 = (4a_n - 3)/(a_n)^2 

Ou seja:
4a_2 - 3 = (4a_1 - 3)/(a_1)^2
4a_3 - 3 = (4a_2 - 3)/(a_2)^2
...
4a_(n+1) - 3 = (4a_n - 3)/(a_n)^2

Multiplicando estas n equacoes e simplificando telescopicamente, obtemos:
4a_(n+1) - 3 = (4a_1 - 3)/(P_n)^2
onde:
P_n = a_1*a_2*...*a_n.

Ou seja:
(P_n)^2 = (4a_1 - 3)/(4a_(n+1) - 3).

Agora eh soh provar que, se a_1 = 3/4 entao:
i) os a_i sao positivos;
e
ii) a_(n+1) - 0 quando n - infinito
que teremos:
P_n - raiz(4a_1 - 3)

[]s,
Claudio.

on 10.03.05 14:16, cleber vieira at [EMAIL PROTECTED] wrote:



cleber vieira [EMAIL PROTECTED] wrote: 
Amigos, gostaria da ajuda de vocês neste problema que na verdade é dividido em três itens, entretanto, os outros dois já foram solucionados e este ainda não consegui resolver.Desde ja muito obrigado.
Os números a_1, a_2 , a_3,... são definidos como segue:
 
a_1 = 3/2 e
 a_(n+1) = [3(a_n)^2 + 4(a_n) - 3]/ 4(a_n)^2.
 
 
Determine lim (a_1)*(a_2)*(a_3)***(a_n)
com n tendendo a infinito.
Ass:Vieira

Yahoo! Acesso Grátis http://br.rd.yahoo.com/mail/taglines/*http://br.acesso.yahoo.com/ - Internet rápida e grátis. Instale o discador do Yahoo! agora.
__
Converse com seus amigos em tempo real com o Yahoo! Messenger 
http://br.download.yahoo.com/messenger/ 






Re: [obm-l] Problema sobre valor minimo

2005-03-10 Por tôpico Claudio Buffara
on 10.03.05 20:27, Marcio M Rocha at [EMAIL PROTECTED] wrote:

 Olá, pessoal.
 
 Leciono Matemática mas não tenho experiência com problemas olímpicos.
 Como penso que todo professor de matemática que se preze deve buscar
 aprender aquilo que não sabe (ao invés de se acomodar à matemática
 burocrática da maioria das escolas), tenho buscado várias fontes.
 Tenho feito algum pequeno progresso, mas vou precisar da ajuda de vocês.
 Tenho certeza de que contarei com ela.
 
 Sendo assim, para começar, peço ajuda com o seguinte problema: Se
 x.y.z.(x + y + z) = 1, qual o valor mínimo de (x + y).(y + z)?
 
 Muito obrigado a todos.
 
 Márcio.
 =
Supondo que x, y e z sao reais positivos, teremos:
xyz(x + y + z) = 1 ==
y^2 + (x+z)y - 1/(xz) = 0 ==
y^2 + (x+z)y + xz - (1/(xz) + xz) = 0 ==
y^2 + (x+z)y + xz = 1/(xz) + xz ==
(x + y)(y + z) = 1/(xz) + xz = 2 quaisquer que sejam x e z positivos, com
igualdade sss xz = 1 ==
(x + y)(y + z) = 2.

O minimo de 2 eh atingido, por exemplo, com x = z = 1 e y = raiz(2) - 1.

[]s,
Claudio.


=
Instruções para entrar na lista, sair da lista e usar a lista em
http://www.mat.puc-rio.br/~nicolau/olimp/obm-l.html
=


Re: [obm-l] Equao

2005-03-10 Por tôpico Claudio Buffara
Title: Re: [obm-l] Equao



Usei sim. Alias, falei uma baita besteira e ninguem me corrigiu...
A equacao tem 2 raizes reais - a outra eh aproximadamente -0,4060952.

Um polinomio de grau 4 nao pode ter apenas uma raiz real (a menos que seja uma raiz dupla, mas nesse caso consideramos que sao duas raizes, mesmo sendo iguais).

[]s,
Claudio.

on 10.03.05 23:42, Alan Pellejero at [EMAIL PROTECTED] wrote:

Legal, Cláudio! Mas uma dúvida...
De onde vc tirou que essa equação do amigo Davidson tem como solução um valor real é, aproximadamente, igual a 1,6917395. Vocë usou programa?
Um abraço!
Alan


Claudio Buffara [EMAIL PROTECTED] wrote:
Polinomio irredutivel sobre um corpo: polinomio que nao pode ser expresso como o produto de dois polinomios nao constantes com coeficientes nesse corpo.
Por exemplo, x^2 + 5x + 6 eh redutivel sobre Q pois eh igual a (x + 2)(x + 3).
Jah x^2 - 2 nao eh redutivel sobre Q, mas eh redutivel sobre R: (x + raiz(2))(x - raiz(2)).
Finalmente, x^2 + 1 nao eh redutivel sobre R mas eh sobre C: (x + i)(x - i).

Existe um teorema, devido a Gauss, que diz que se um polinomio com coeficientes inteiros eh redutivel sobre Q, entao ele eh redutivel sobre Z, ou seja, se f(x) pertence a Z[x] e f(x) = p(x)*q(x) com p(x) e q(x) em Q[x], entao existem polinomios p1(x) e q1(x) em Z[x] tais que f(x) = p1(x)*q1(x).

O criterio de Eisenstein diz o seguinte: dado f(x) = a_0 + a_1*x + ... + a_(n-1)*x^(n-1) + a_n*x^n em Z[x], 
se existe um primo p tal que: 
p divide a_0, a_! 1, ..., a_(n-1), 
p nao divide a_n, e
p^2 nao divide a_0,
entao f(x) eh irredutivel sobre Q.

As demonstracoes desses dois resultados nao sao muito dificeis mas sao um belo exercicio de teoria elementar dos numeros.

[]s,
Claudio.

on 10.03.05 08:07, Alan Pellejero at [EMAIL PROTECTED] wrote:

Caro amigo Cláudio...Se possível, gostaria de saber o que é um polinômio irredutível por Eisentein com p=2...Um grande abraço! 

x^4 + 2x^3 - 2x^2 - 6x - 2 = 0 ==
polinomio irredutivel por Eisenstein com p = 2 

Davidson Lima [EMAIL PROTECTED] wrote: 
 Não meu amigo, a equação é x^2+1/(x+1)^2=3. 

 Felicidades. 

 Davidson Estanislau

rt Smith [EMAIL PROTECTED] wrote:

From: Qwert Smith [EMAIL PROTECTED]
Date: Wed, 09 Mar 2005 23:08:06 -0500
To: obm-l@mat.puc-rio.br
Subject: Re: [obm-l] Equao

Sera que nao faltou um parentesis no numerador?

(x^2 + 1)/(x+1)^2=3


From: Claudio Buffara [EMAIL PROTECTED]

Fazendo o obvio ==
x^4 + 2x^3 + x^2 + 1 = 3x^2 + 6x + 3 ==
x^4 + 2x^3 - 2x^2 - 6x - 2 = 0 ==
polinomio irredutivel por Eisenstein com p = 2 ==
nao vejo nenhuma solucao bonitinha

Essa equacao tem uma unica raiz real, igual a aproximadamente 1,6917395.

[]s,
Claudio.

on 09.03.05 18:35, Davidson Lima at [EMAIL PROTECTED] wrote:


 Meus amigos, como fao para! resolver a questo:

 x^2+1/(x+1)^2=3

 Desde j agradeo a ateno.

 Davidson Estanislau



=
Instrues para entrar na lista, sair da lista e usar a lista em
http://www.mat.puc-rio.br/~nicolau/olimp/obm-l.html
=




The top resources for math --- http://www.Math.com/
Instruções para entrar na lista, sair da lista e usar a lista em http://www.mat.puc-rio.br/~nicolau/olimp/obm-l.html 


Yahoo! Acesso Grátis http://br.rd.yahoo.com/mail/taglines/*http://br.acesso.yahoo.com/ - Internet rápida e grátis. Instale o discador do Yahoo! agora. 



Yahoo! Mail  http://us.rd.yahoo.com/mail/br/taglines/*http://mail.yahoo.com.br/ - Com 250MB de espaço. Abra sua conta! 






Re: [obm-l] Equação

2005-03-09 Por tôpico Claudio Buffara
Title: Re: [obm-l] Equação



Fazendo o obvio ==
x^4 + 2x^3 + x^2 + 1 = 3x^2 + 6x + 3 ==
x^4 + 2x^3 - 2x^2 - 6x - 2 = 0 ==
polinomio irredutivel por Eisenstein com p = 2 ==
nao vejo nenhuma solucao bonitinha

Essa equacao tem uma unica raiz real, igual a aproximadamente 1,6917395.

[]s,
Claudio.

on 09.03.05 18:35, Davidson Lima at [EMAIL PROTECTED] wrote:


 Meus amigos, como faço para resolver a questão: 

 x^2+1/(x+1)^2=3 

 Desde já agradeço a atenção. 

 Davidson Estanislau






Re: [obm-l] Equação

2005-03-09 Por tôpico Claudio Buffara
Nesse caso eu nao sei o que eh mais deprimente numa lista que trata de
olimpiadas de matematica: alguem escrever x^2+1/(x+1)^2 quando queria dizer
(x^2+1)/(x+1)^2 ou alguem nao saber resolver uma misera equacao do 2o. grau.


on 10.03.05 01:08, Qwert Smith at [EMAIL PROTECTED] wrote:

 Sera que nao faltou um parentesis no numerador?
 
 (x^2 + 1)/(x+1)^2=3
 
 
 From: Claudio Buffara [EMAIL PROTECTED]
 
 Fazendo o obvio ==
 x^4 + 2x^3 + x^2 + 1 = 3x^2 + 6x + 3 ==
 x^4 + 2x^3 - 2x^2 - 6x - 2 = 0 ==
 polinomio irredutivel por Eisenstein com p = 2 ==
 nao vejo nenhuma solucao bonitinha
 
 Essa equacao tem uma unica raiz real, igual a aproximadamente 1,6917395.
 
 []s,
 Claudio.
 
 on 09.03.05 18:35, Davidson Lima at [EMAIL PROTECTED] wrote:
 
 
 Meus amigos, como faço para resolver a questão:
 
 x^2+1/(x+1)^2=3
 
 Desde já agradeço a atenção.
 
 Davidson Estanislau
 
 
 
 =
 Instruções para entrar na lista, sair da lista e usar a lista em
 http://www.mat.puc-rio.br/~nicolau/olimp/obm-l.html
 =
 


=
Instruções para entrar na lista, sair da lista e usar a lista em
http://www.mat.puc-rio.br/~nicolau/olimp/obm-l.html
=


Re: [obm-l] Racional e Irracional

2005-03-05 Por tôpico Claudio Buffara
Title: Re: [obm-l] Racional e Irracional



Ainda nao ficou claro se o que voce quer eh provar que:
a) arccos(3/5) eh irracional quando expresso em radianos
ou
b) arccos(3/5) eh irracional quando expresso em graus (ou seja, eh um multiplo irracional de Pi).

Ambas as afirmativas sao verdadeiras mas, como o Nicolau mostrou, (b) tem uma demonstracao compativel com o nivel medio enquanto que (a) depende de um teorema de nivel universitario.

[]s,
Claudio.

on 05.03.05 11:23, cfgauss77 at [EMAIL PROTECTED] wrote:

 Gostaria de agradecer a atenção dipensada pelos colegas sobre arccos(3/5) ser irracional. Caso alguém consiga uma forma mais elementar nessa demonstração ficaria agradecido se me enviasse, gostaria de apresentar esse problema numa turma de ensino médio preparatoria para IME e ITA e ainda não sei se, da forma que me foi proposto, alguém conseguiria entender. Ficaria grato também por opiniões. No mais, vou passar o final de semana dando uma uma olhada no livro do Djairo na tentativa de conseguir algo um pouco melhor.
 
 Até a próxima







Re: [obm-l] séries 2

2005-03-04 Por tôpico Claudio Buffara
on 02.03.05 19:57, Demetrio Freitas at [EMAIL PROTECTED]
wrote:

 
 Agora um difícil:
 
 Calcule o valor para onde converge a soma:
 
 S[n]= +1 -1/(1+1) +1/(1+4) -1/(1+9) +1/(1+16)
 -1/(1+25)
 +1/(1+36)...
 
 Isto é:
 Sinais - + - + - + - + -...
 Denominador - 1+n^2, com n(0,oo): 1, 2, 5, 10, 17,
 26, 37, 50, 65, 82, 101...
 
 []´s
 
 Demétrio 
 
 
Isso eh a serie de Fourier do cosseno hiperbolico.

cosh(x) = 
(2*senh(Pi)/Pi)*(1/2 - cos(x)/(1+1^2) + cos(2x)/(1+2^2) - cos(3x)/(1+3^2) +
...).

Dai, com x = 0, fica:
1 = (2*senh(Pi)/Pi)*(1/2 - 1/(1+1^2) + 1/(1+2^2) - 1/(1+3^2) + ...) ==

1 = (2*senh(Pi)/Pi)*(S - 1/2), onde S eh o valor da sua serie ==

S = Pi/(2*senh(Pi)) + 1/2 = Pi/(e^Pi - e^(-Pi)) + 1/2

[]s,
Claudio.



=
Instruções para entrar na lista, sair da lista e usar a lista em
http://www.mat.puc-rio.br/~nicolau/olimp/obm-l.html
=


Re: [obm-l] séries - CORRECAO (II)

2005-03-04 Por tôpico Claudio Buffara
on 02.03.05 22:36, Claudio Buffara at [EMAIL PROTECTED] wrote:

 Ops! Esqueci do logaritmo nas 3 ultimas linhas.
 Acho que agora tah certo.
 
 --
 From: Claudio Buffara [EMAIL PROTECTED]
 Date: Wed, 02 Mar 2005 22:26:29 -0300
 To: obm-l@mat.puc-rio.br
 Subject: Re: [obm-l] séries
 
 on 02.03.05 19:50, Demetrio Freitas at [EMAIL PROTECTED]
 wrote:
 
 
 Saudações,
 
 Um de séries, facilzinho para esquentar:
 
 Calcule o valor para onde converge a soma:
 
 S[n]= 1 +2/3 +1/5 -1/7 -2/9 -1/11 +1/13 +2/15 +1/17
 -1/21 -2/23 -1/25 +1/27 +2/29 ...
 
 Isto é:
 numerador- 1 2 1 1 2 1 1 2 1 1...
 sinais   - + + + - - - + + + -...
 
 []´s
 
 Demétrio
 
 
 Considere as sequencias (A_n) e (B_n), dadas por:
 A_n = 2*cos(n*Pi/3 - Pi/6)/(2n - 1)
 e
 B_n = 2*sen(n*Pi/3 - Pi/6)/(2n - 1)
 
 Queremos o valor de S = SOMA A_n.
 
 A_n + i*B_n = 
 2*exp(i*(n*Pi/3 - Pi/6))/(2n - 1) =
 2*exp(i*(2n-1)*Pi/6)/(2n - 1) =
 2*(exp(i*Pi/6))^(2n-1)/(2n - 1) ==
 
 SOMA (A_n + i*B_n) =
 2*SOMA (exp(i*Pi/6))^(2n-1)/(2n-1) =
 2*(1/2)*log((1 + exp(i*Pi/6))/(1 - exp(i*Pi/6))) =
 log(i*sen(Pi/6)/(1 - cos(Pi/6))) =
 log(i*(2 + raiz(3))) =
 i*(Pi/2 + 2*k*Pi) + log(2 + raiz(3)) ==
 
 S = SOMA A_n = log(2 + raiz(3)).
 
 []s,
 Claudio.
 
 
Depois da msg do Demetrio dizendo que essa era uma serie de Pi eu me dei
conta de que falei besteira acima.

Os numeradores 1, 2, 1, -1, -2, -1, ... para n = 1, 2, 3, 4, 5, 6, ... sao
justamente os valores de 2*sen(n*Pi/3 - Pi/6).

Ou seja, a serie dele nao eh SOMA A_n mas sim SOMA B_n = valor principal da
parte imaginaria de log(i*(2 + raiz(3))) = Pi/2.

[]s,
Claudio.


=
Instruções para entrar na lista, sair da lista e usar a lista em
http://www.mat.puc-rio.br/~nicolau/olimp/obm-l.html
=


Re: [obm-l] questao de regra de tres composta

2005-03-04 Por tôpico Claudio Buffara
Oi, pessoal:

Nesses problemas, eu raciocino assim:

Quanto mais trabalhadores, mais dias de trabalho e mais horas trabalhadas
por dia, mais trabalho eh realizado.

Assim, eh razoavel que se tenha T = k*M*D*H, onde:
T = quantidade de trabalho realizada;
M = no. de trabalhadores (M para Mao de obra);
D = no. de dias de trabalho;
H = no. de horas trabalhadas por dia;
k = constante de proporcionalidade a ser determinada.

Do enunciado, temos 2/5 = k*24*10*7 == k = 1/4200.
 
Portanto, quando T = 3/5 (trabalho restante), M = 20 (= 24 - 4) e H = 6,
teremos:
3/5 = (1/4200)*20*D*6 == D = 21 dias.

[]s,
Claudio.

on 04.03.05 14:44, Qwert Smith at [EMAIL PROTECTED] wrote:

 
 Esta correto?
 Vamos supor que nada mudou... ou seja a jornada de trabalho continua a mesma
 e o numero de trabalhadores tb.  Como 2/5 do trablho nessas condicoes levou
 10 dias, precisariamos de mais 15 dias pra terminar o servico.  Voce acha
 mesmo que diminuir os trabalhores e a carga horaria faz o trabalho acabar
 mais rapido?
 
 From: Brunno [EMAIL PROTECTED]
 Reply-To: obm-l@mat.puc-rio.br
 To: obm-l@mat.puc-rio.br
 Subject: Re: [obm-l] questao de regra de tres composta
 Date: Fri, 4 Mar 2005 13:37:41 -0300
 
 Esta correto, mas poderia mostrar como chegou a essa resposta
 um abraco
 
 - Original Message -
 From: elton francisco ferreira [EMAIL PROTECTED]
 To: obm-l@mat.puc-rio.br
 Sent: Friday, March 04, 2005 11:17 AM
 Subject: Re: [obm-l] questao de regra de tres composta
 
 
 14 dias. se estiver errado, favor informar.
 --- [EMAIL PROTECTED] wrote:
 Ola pessoal poderiam me ajudar nesta questao
 Se 2/5 de um trabalho foram feitos em 10 dias por
 24 operários que
 trabalhavam 7 horas por dia, então quantos dias
 serão necessários para
 terminar o trabalho, sabendo que 4 operários foram
 dispensados e que os
 restantes agora trabalham 6 horas por dia?
 
 Um abraco
 
 


=
Instruções para entrar na lista, sair da lista e usar a lista em
http://www.mat.puc-rio.br/~nicolau/olimp/obm-l.html
=


Re: [obm-l] Racional ou Irracional???

2005-03-04 Por tôpico Claudio Buffara
OK. Falha minha.

No mais, pra quem quiser, aqui estah uma demonstracao do teorema de
Lindemann, a que voce se referiu na msg anterior:
http://www.math.sc.edu/~filaseta/gradcourses/Math785/Math785Notes7.pdf

Alias, estas notas de aula sobre numeros irracionais e transcendentes sao
bem interessantes (pelo menos as partes que eu consegui entender),
especialmente pra quem nao tem o livro do Niven. Nelas voce tambem pode
encontrar a demonstracao de que a sequencia das partes fracionarias de n*a
(n inteiro e a irracional) sao uniformemente distribuidas em [0,1], o que
estende o resultado jah discutido aqui na lista de que ela eh densa nesse
intervalo.

[]s,
Claudio.

on 04.03.05 10:11, Nicolau C. Saldanha at [EMAIL PROTECTED] wrote:

 On Thu, Mar 03, 2005 at 12:41:00PM -0300, Claudio Buffara wrote:
 Corolário do corolário:
 Se x é racional, x diferente de 0, então cos(x) é irracional.
 Se x é racional, x diferente de 1, então arccos(x) é irracional.
 
 Tem algumas excecoes, tais como r = 0, 1/2, -1/2, 1 e -1.
 Nesses casos, arccos(r) e arcsen(r) sao multiplos racionais de Pi, mas acho
 que essas sao as unicos excecoes.
 
 Acho que não estamos nos entendendo.
 
 Se r = 1/2 então arccos(r) é de fato múltiplo racional de Pi
 mas não era disso que eu estava falando na segunda parte da mensagem
 nem foi esta a pergunta original.
 
 O que eu disse é que arccos(r) é irracional para r racional,
 r diferente de 1, o que é correto exatamente como eu enunciei,
 sem outras exceções além de r=1. Por exemplo, arccos(0) = pi/2
 é irracional pois pi é irracional.
 
 Se a pergunta for para quais racionais r temos que arccos(r)
 é um múltiplo racional de pi então a resposta é que isto ocorre
 exatamente para os valores que você listou: 0, +-1 e +-1/2.
 A prova disso é bem simples. Se x é racional então
 2cos(pi x) = exp(i pi x) + exp(-i pi x) é um inteiro algébrico.
 Assim se cos(pi x) for racional, 2 cos(pi x) deve ser racional
 e inteiro algébrico, logo inteiro. Como -2 = 2 cos(pi x) = 2
 devemos ter cos(x) = 0, +-1 ou +-1/2.
 
 []s, N.


=
Instruções para entrar na lista, sair da lista e usar a lista em
http://www.mat.puc-rio.br/~nicolau/olimp/obm-l.html
=


Re: [obm-l] Racional ou Irracional???

2005-03-03 Por tôpico Claudio Buffara
on 03.03.05 11:21, Nicolau C. Saldanha at [EMAIL PROTECTED] wrote:

 On Thu, Mar 03, 2005 at 07:44:53AM -0300, cfgauss77 wrote:
 Gostaria de uma ajuda no seguinte problema ou que me indicassem uma
 literatura que me ajudasse no assunto.
 
 Demonstre que arcCos(3/5) é irracional.
 
 Posso afirmar corretamente que arcCos(r) ou arcSen(r), com r racional, sempre
 será um irracional?
 
 Será que você não quer dizer que arccos(3/5)/pi é irracional?
 Se for isto, o seu problema é uma reformulação de uma questão
 que caiu no vestibular do IME de 1980-81 e que já foi discutida
 aqui mais de uma vez. O enunciado original era mais ou menos o seguinte:
 
 Seja z = (3+4i)/5. Prove que z não é raiz da unidade, ou seja,
 que z^n não é igual a 1 para nenhum n.
 
 A solução mais curta e elementar que eu conheço é a seguinte.
 Define z^n = (an + bn i)/(5^n). Assim
 a0 = 1, b0 = 0
 a1 = 3, b1 = 4
 a2 = -7, b2 = 24
 ...
 a(n+1) = an^2 - bn^2, b(n+1) = 2 an bn
 
 Agora é fácil provar por indução que an = 3 (mod 5) e bn = 4 (mod 4)
 para todo n = 1. Em particular, bn é diferente de 0 e z^n é diferente de 1.
 
 O problema como você escreveu também é correto mas eu não sei fazer
 de forma elementar (mas também não tentei muito). Ele é um corolário
 do teorema de Lindemann:
 
 Teorema: Sejam a1, a2, ..., am números algébricos distintos.
 Então exp(a1), exp(a2), ..., exp(am) são linearmente independentes
 sobre o corpo dos números algébricos.
 
 (Estou traduzindo de Irrational Numbers, Ivan Niven,
 The Carus Mathematical Monographs, no. 11, MAA, capítulo 9.)
 
 Corolário: Se x é algébrico, x diferente de 0, então cos(x) é transcendente.
 
 Demonstração: Sejam a1 = 0, a2 = ix, a3 = -ix.
 Temos cos(x) = (exp(a2) + exp(a3))/2. Se cos(x) fosse algébrico,
 seria um múltiplo algébrico de 1 = exp(a1), contrariando
 o teorema de Lindemann.
 
 Corolário do corolário:
 Se x é racional, x diferente de 0, então cos(x) é irracional.
 Se x é racional, x diferente de 1, então arccos(x) é irracional.
 
 []s, N.

Tem algumas excecoes, tais como r = 0, 1/2, -1/2, 1 e -1.
Nesses casos, arccos(r) e arcsen(r) sao multiplos racionais de Pi, mas acho
que essas sao as unicos excecoes.

[]s,
Claudio.


=
Instruções para entrar na lista, sair da lista e usar a lista em
http://www.mat.puc-rio.br/~nicolau/olimp/obm-l.html
=


Re: [obm-l] Racional ou Irracional???

2005-03-03 Por tôpico Claudio Buffara
Title: Re: [obm-l] Racional ou Irracional???



De uma olhada nas 3 primeiras paginas deste pdf:
http://www.math.sc.edu/~filaseta/gradcourses/TheMath784Notes.pdf

[]s,
Claudio.

on 03.03.05 07:44, cfgauss77 at [EMAIL PROTECTED] wrote:

 Gostaria de uma ajuda no seguinte problema ou que me indicassem uma literatura que me ajudasse no assunto.
 
 Demonstre que arcCos(3/5) é irracional.
 
Posso afirmar corretamente que arcCos(r) ou arcSen(r), com r racional, sempre será um irracional?
 
 Desde já agradeço a cooperação!!! 







Re: [obm-l] séries

2005-03-02 Por tôpico Claudio Buffara
on 02.03.05 19:50, Demetrio Freitas at [EMAIL PROTECTED]
wrote:

 
 Saudações,
 
 Um de séries, facilzinho para esquentar:
 
 Calcule o valor para onde converge a soma:
 
 S[n]= 1 +2/3 +1/5 -1/7 -2/9 -1/11 +1/13 +2/15 +1/17
 -1/21 -2/23 -1/25 +1/27 +2/29 ...
 
 Isto é:
 numerador- 1 2 1 1 2 1 1 2 1 1...
 sinais   - + + + - - - + + + -...
 
 []´s
 
 Demétrio
 
 
Considere as sequencias (A_n) e (B_n), dadas por:
A_n = 2*cos(n*Pi/3 - Pi/6)/(2n - 1)
e
B_n = 2*sen(n*Pi/3 - Pi/6)/(2n - 1)

Queremos o valor de S = SOMA A_n.

A_n + i*B_n = 
2*exp(i*(n*Pi/3 - Pi/6))/(2n - 1) =
2*exp(i*(2n-1)*Pi/6)/(2n - 1) =
2*(exp(i*Pi/6))^(2n-1)/(2n - 1) ==

SOMA (A_n + i*B_n) =
2*SOMA (exp(i*Pi/6))^(2n-1)/(2n-1) =
2*(1/2)*log((1 + exp(i*Pi/6))/(1 - exp(i*Pi/6))) =
i*sen(Pi/6)/(1 - cos(Pi/6)) =
i/(2 - raiz(3)) =
imaginario puro ==

Se eu nao errei nenhuma conta, S = SOMA A_n = 0

[]s,
Claudio.


=
Instruções para entrar na lista, sair da lista e usar a lista em
http://www.mat.puc-rio.br/~nicolau/olimp/obm-l.html
=


[obm-l] séries - CORRECAO

2005-03-02 Por tôpico Claudio Buffara
Ops! Esqueci do logaritmo nas 3 ultimas linhas.
Acho que agora tah certo.

--
From: Claudio Buffara [EMAIL PROTECTED]
Date: Wed, 02 Mar 2005 22:26:29 -0300
To: obm-l@mat.puc-rio.br
Subject: Re: [obm-l] séries

on 02.03.05 19:50, Demetrio Freitas at [EMAIL PROTECTED]
wrote:

 
 Saudações,
 
 Um de séries, facilzinho para esquentar:
 
 Calcule o valor para onde converge a soma:
 
 S[n]= 1 +2/3 +1/5 -1/7 -2/9 -1/11 +1/13 +2/15 +1/17
 -1/21 -2/23 -1/25 +1/27 +2/29 ...
 
 Isto é:
 numerador- 1 2 1 1 2 1 1 2 1 1...
 sinais   - + + + - - - + + + -...
 
 []´s
 
 Demétrio
 
 
Considere as sequencias (A_n) e (B_n), dadas por:
A_n = 2*cos(n*Pi/3 - Pi/6)/(2n - 1)
e
B_n = 2*sen(n*Pi/3 - Pi/6)/(2n - 1)

Queremos o valor de S = SOMA A_n.

A_n + i*B_n = 
2*exp(i*(n*Pi/3 - Pi/6))/(2n - 1) =
2*exp(i*(2n-1)*Pi/6)/(2n - 1) =
2*(exp(i*Pi/6))^(2n-1)/(2n - 1) ==

SOMA (A_n + i*B_n) =
2*SOMA (exp(i*Pi/6))^(2n-1)/(2n-1) =
2*(1/2)*log((1 + exp(i*Pi/6))/(1 - exp(i*Pi/6))) =
log(i*sen(Pi/6)/(1 - cos(Pi/6))) =
log(i*(2 + raiz(3))) =
i*(Pi/2 + 2*k*Pi) + log(2 + raiz(3)) ==

S = SOMA A_n = log(2 + raiz(3)).

[]s,
Claudio.


=
Instruções para entrar na lista, sair da lista e usar a lista em
http://www.mat.puc-rio.br/~nicolau/olimp/obm-l.html
=


Re: [obm-l] Re: [obm-l] Probleminha de F ísica (o de matemática)

2005-03-01 Por tôpico Claudio Buffara
Isso eh a soma de Riemann de I = Integral(1..3) dx/x = log(3).
Fazendo a subdivisao do intervalo [1,3] em 2N sub-intervalos de comprimento
1/N cada, teremos: 
deltax = (3-1)/(2N) = 1/N
e  
x_k = 1 + k*deltax = 1 + k/N = (N + k)/N para 0 = k = 2N-1.

Logo: 
I = lim(N - infinito) SOMA(k=0 a 2N-1) (1/x_k)*deltax =
lim(N - infinito) SOMA(k = 0 a 2N-1) 1/(N + k) =
lim(N - infinito) (1/N + 1/(N+1) + ... + 1/(3N-1)).

[]s,
Claudio.

on 01.03.05 13:06, Daniel Nunes at [EMAIL PROTECTED] wrote:

 Não é difícil provar que valem
 1/x  log(x+1) - log(x) = log((x + 1)/x) e
 1/x  log(x) - log(x-1) = log(x/(x - 1))
 para todo x =2.
 
 Definindo
 B_n = log((n + 1)/n) + ... + log((3n - 1)/(3n - 2))
 = log((n+1)*(n+2)*...*(3n - 1)/[n*(n+1)*...*(3n - 2)])
 = log[(3n - 1)/n] = log[3 - 1/n]
 
 e
 
 C_n = log(n/(n - 1)) + ... + log((3n - 2)/(3n - 3))
 = log((n*(n+1)*...*(3n - 2)/[(n - 1)*n*...*(3n - 3)])
 = log[(3n - 2)/(n - 1)] = log[3 - 1/(n-1)]
 
 temos
 B_n  A_n   C_n para todo n =2, logo se B, A e C são
 os respectivos limites quando n -- +oo, vale
 B = A = C. Obviamente, B = C = log(3), e portanto
 A_n -- log(3).
 
 []s,
 Daniel
 
 
 
 - Original Message -
 From: Paulo Santa Rita [EMAIL PROTECTED]
 To: obm-l@mat.puc-rio.br
 Sent: Tuesday, March 01, 2005 9:05 AM
 Subject: [obm-l] RE: [obm-l] Probleminha de Física
 
 
 Essa e uma lista de Matematica. Fazendo justica a isso, aqui vai um
 problema
 de Matematica :
 
 Seja An=1/N + 1/(N+1) + ... 1/(3N-3) + 1/(3N-2). Calcule lim An, quando N
 tende ao infinito.
 
 


=
Instruções para entrar na lista, sair da lista e usar a lista em
http://www.mat.puc-rio.br/~nicolau/olimp/obm-l.html
=


Re: [obm-l] En: [obm-l] Tangência...

2005-03-01 Por tôpico Claudio Buffara
on 01.03.05 23:14, Vinícius Meireles Aleixo at [EMAIL PROTECTED]
wrote:

 Eu gostaria de saber qual é o conceito rigoroso de reta tangente a uma
 curva
 qualquer (circunferência, elipse, hipérbole, parábola, etc...)
 
 A tangente é a linha ou superfície que toca outra linha ou superfície em um
 só ponto.
 Cara, acho essa definição bem esclarecedora e concisa.Caso alguém tenha + a
 dizer...
 
 
 abraços
 
 Vinícius Meireles Aleixo
 
Logo, o eixo y eh tangente aa parabola y = x^2 e tambem ao eixo x.
Alem disso, duas retas concorrentes sao tangentes uma a outra.


=
Instruções para entrar na lista, sair da lista e usar a lista em
http://www.mat.puc-rio.br/~nicolau/olimp/obm-l.html
=


Re: [obm-l] Probleminha de Física

2005-02-28 Por tôpico Claudio Buffara
Title: Re: [obm-l] Probleminha de Física



Se o corpo estah em equilibrio (ou seja, em repouso ou em MRU) no referencial do observador, entao a forca resultante sobre ele eh nula. Em particular, a componente horizontal da forca resultante eh nula.
Ou seja, Forca Resultante (horizontal) = F - Fat = 20N - Fat = 0 == Fat = 20 N.

O enunciado estah mal feito pois nao deixa claro se o corpo estah ou nao em equilibrio.

Repare que o modulo da forca de atrito soh seria igual ao coeficiente de atrito estatico vezes o modulo da forca normal se o corpo estivesse na iminencia de entrar em movimento. Acho que essa eh a pegadinha do problema, pois nesse caso, a forca de atrito valeria 0,25*10*10 = 25 N, e o candidato desatento marcaria opcao (b).

[]s,
Claudio.

\ 
on 28.02.05 11:26, Alan Pellejero at [EMAIL PROTECTED] wrote:

Olá amigos, não consigo entender o porquê da resposta ser a a. Alguém me ajuda???Obrigado!!! 

 

3) Uma força de 20N é aplicada a um corpo de massa 10Kg que está apoiado sobre uma superfície horizontal, cujo coeficiente de atrito estático entre o corpo e a superfície é de 0,25. Qual a força de atrito, em módulo exercida pela superfície. 

Dada g = 10m/s2 

a) 20N 

b) 25N 

c) 5N 

d) 17,5N

Yahoo! Acesso Grátis http://br.rd.yahoo.com/mail/taglines/*http://br.acesso.yahoo.com/ - Internet rápida e grátis. Instale o discador do Yahoo! agora. 






[obm-l] Conicas, Triangulos e Divisores

2005-02-28 Por tôpico Claudio Buffara
O problema do Bruno sobre o angulo reto na elipse eh muito interessante e,
de fato, eu nao consegui resolve-lo usando apenas geometria sintetica.
Alias, eu nao conheco nenhuma referencia bibliografica sobre o assunto em
portugues. Alguem sabe de algum livro ou artigo em portugues que trate de
secoes conicas sem ser por geometria analitica?

***

O problema do Rogerio sobre o triangulo equilatero inscrito num triangulo
continua sem solucao.

***

No mais, aqui vai um bonitinho de divisibilidade:
Qual o maior inteiro que eh divisivel por todos os inteiros positivos
menores do que sua raiz quadrada?

[]s,
Claudio.

=
Instruções para entrar na lista, sair da lista e usar a lista em
http://www.mat.puc-rio.br/~nicolau/olimp/obm-l.html
=


Re: [obm-l] Denovo o problema de elipse...

2005-02-25 Por tôpico Claudio Buffara
Espero sinceramente que voce esteja de brincadeira...

[]s,
Claudio.

on 25.02.05 18:54, Bruno Bruno at [EMAIL PROTECTED] wrote:

 Os angulos BFT' e AFT são complementares, se e somente se o angulo
 TFT' for reto. Como FBT' e FAT são retos, concluimos que os triangulos
 BFT' e AFT são semelhantes se e somente se o angulo TFT'. Vamos chamas
 o angulo BFT' de m e o anfulo AFT de n. T'B será b , FB será a, TA
 será d e AF será c.
 
 Assim, sen m = b/a = c/d
 Por semelhança de triangulos, d/a = b/c
 Formamos entao um sistema:
 {b/a = c/d
 {d/a = c/b
 Tal sistema só será valido, se os triangulos forem semelhantes, o que
 ocorrerá somente no caso de TFT' ser reto. Para testar o sistema,
 isolamos c na primeira linha e temos:
 c = bd/a
 d/a = c/b = (bd/a)/b = d/a
 
 Ora, como d/a = d/a se e somente se TFT' for reto, TFT' é reto.
 
 
 
 
 On Fri, 25 Feb 2005 17:55:39 -0300, Bruno Bonagura [EMAIL PROTECTED]
 wrote:
 Estou mandando novamente um problema que mandei para a lista há um tempo
 atrás. Imagino que os senhores tiveram dificuldade em acessar a imagem pois
 o servidor do uol não permite acesso direto a arquivos de imagem.
 
 Está aqui o link do enunciado.
 http://cienciasexatas.sites.uol.com.br/elipse.htm
 
 Gostaria de uma demonstração com uso de geometria plana. Através de
 analítica eu ja consegui a prova mas gostaria muito de ter uma demonstração
 através de conceitos da geometria euclidiana.
 
 Agradeço respostas!
 
 Bruno Bonagura
 http://cienciasexatas.blog.uol.com.br
 
 =
 Instruções para entrar na lista, sair da lista e usar a lista em
 http://www.mat.puc-rio.br/~nicolau/olimp/obm-l.html
 =
 


=
Instruções para entrar na lista, sair da lista e usar a lista em
http://www.mat.puc-rio.br/~nicolau/olimp/obm-l.html
=


Re: [obm-l] Saida de emergencia

2005-02-24 Por tôpico Claudio Buffara
Title: Re: [obm-l] Saida de emergencia



http://mathworld.wolfram.com/BrachistochroneProblem.html

[]s,
Claudio.

on 24.02.05 14:00, Rogerio Ponce at [EMAIL PROTECTED] wrote:

Uma equipe da Boeing deseja construir uma rampa (um escorregador) para a saida de emergencia de um aviao com as seguintes caracteristicas:
- a saida de emergencia esta' a 10m de altura em relacao ao chao.
- a rampa termina a 10m de distancia da projecao vertical da porta do aviao
- a aceleracao da gravidade e' 10m/s^2 , e a rampa nao oferece atrito
 
Sabendo-se que a descida pelo escorregador deve ser feita no menor tempo possivel, e que o passageiro nao da' nenhum impulso na saida (isto e', a velocidade inicial e' zero) , qual a funcao que descreve o perfil da rampa?
 
Abracos,
Rogerio Ponce
 






Re: [obm-l] O FANTASMA DA MÁQUINA!

2005-02-24 Por tôpico Claudio Buffara
on 24.02.05 12:00, Chicao Valadares at [EMAIL PROTECTED] wrote:
 
 Quanto ao probleminha da calculadora do feirante
 fica como desafio de
 despedida devido sua resolução ser muito extensa.
 Esqueçam a calculadora
 científica e divirtam-se!
 
Ou seja, o que se pede eh expressar a*b usando apenas as operacoes de adicao
e de se tomar o inverso (1/x).

Tenho quese certeza de que isso jah foi resolvido aqui na lista.

[]s,
Claudio.


=
Instruções para entrar na lista, sair da lista e usar a lista em
http://www.mat.puc-rio.br/~nicolau/olimp/obm-l.html
=


Re: [obm-l] Re- listinha boa

2005-02-24 Por tôpico Claudio Buffara
Title: Re: [obm-l] Re- listinha boa



A equacao imediatamente anterior a (7) (ou seja, H = ) dah o valor da altura atingida por um estilhaco a uma distancia horizontal de D/2 da explosao em funcao da tangente do angulo w de lancamento.
Eh uma funcao da forma y = a*tg^2(w) + b*tg(w) + c, com a = - (g*D^2)/(8*Vo^2)  0. 
Logo, o valor maximo eh dado por -delta/(4a) quando tg(w) = -b/2a.
Assim, basta fazer H = -delta/(4a) e resolver esta inequacao para D.

[]s,
Claudio.

on 24.02.05 09:14, Murilo Rebouças Fernandes de Lima at [EMAIL PROTECTED] wrote:

1)As provas de um detonador de granadas efetuam-se no cemtro do
fundo de um poço cilindrico de profundidade H.Os estilhaços da
granada, que se produzem depois da explosão e cujas velocidades não
ultrapassam Vo, não devem cair na superfície da terra.Qual deverá
ser o diametro minimo d do poço?
 
 
Suponha que um estilhaço sai com velocidade inicial que forma um
ângulo w com o plano do fundo do poço. Sejam x e y os
deslocamentos horizontal e vertical nos pontos onde o estilhaço está acima da
superfície da terra. Considere ainda o plano cartesiano com origem no centro do fundo do poço.
g a aceleracao da gravidade.
 
 
(1) x = v0 * (cos w) * t
(2) y = v0 * (sen w) * t - (g * t^2 )/2
 
(3) fazendo (x,y) a borda do poço temos o par (D/2,H)  
 
de (1) e (3) temos:
 
(4) t = D / (2 * v0 * (cos w))
 
de (2) , (3) e (4) temos:
 
H = (sen w) * D / (2 * (cos w)) - g * D^2 / (8 * (v0^2) * (cos w)^2)
 
(5) H = D * (tan w) / 2 - g * D^2 * ((sec w)^2) / (8 * (v0^2))
 
(6) (sec w)^2 = 1 + (tan w)^2
 
de (5) e (6)
 
H = D * (tan w) / 2 - g * D^2 * (1 + (tan w)^2) / (8 * (v0^2))
 
(7) { g*D^2/(8*(v0^2)) } * (tan w)^2 - {D/2} * (tan w) + { H+g*D^2/(8 * (v0^2)) } = 0
 
(7) é uma equacao do segundo grau em funcao de w. Para que D tenha o valor minimo é necessário que w (o argumento) tenha valor unico ou seja: raiz dupla. Delta = 0.
 
Fazendo as cxontas do delta e isolando D temos:
 
D = 2*v0*sqrt((v0/g)^2 - 2*H/g )
 
D = 2 v0 sqrt( (v0/g)^2 - 2H/g )
 
Talvez a parte do Delta ou alguma continha esteja errada pq to com pressa. Confiram ai. 
 
Abraços,
 
Murilo.
 
 
 
 
- Original Message - 
From: Vinícius Meireles Aleixo mailto:[EMAIL PROTECTED] 
To: obm-l@mat.puc-rio.br 
Sent: Wednesday, February 23, 2005 10:55 PM
Subject: [obm-l] Re- listinha boa


 1)As provas de um detonador de granadas efetuam-se no cemtro do
 fundo de um poço cilindrico de profundidade H.Os estilhaços da
 granada, que se produzem depois da explosão e cujas velocidades não
 ultrapassam Vo, não devem cair na superfície da terra.Qual deverá
 ser o diametro minimo d do poço?
 
Suponha que um estilhaço sai com velocidade inicial que forma um
ângulo w com o plano do fundo do poço. Sejam x_1 e x_2 os
deslocamentos horizontais nos pontos onde o estilhaço está acima da
superfície da terra. Seja ainda r = d/2.
 
[...]
 
Chamando k^2 de 2*g*H/v_0^2, o nosso problema se reduz a achar o máximo
de
 
cos w*[sen w + sqrt(sen^2 w - k^2)]. Esse máximo será o valor de r.
Como 0  w  pi/2, os extremos não maximizam a função e cos w = sqrt(1
- sen^2 w). Chamando sen w de u (logo 0  u  1), temos que maximizar
 
sqrt(1 - u^2)*(u + sqrt(u^2 - k^2)).
 
Apesar que eu não fiz a conta, não parece ser muito fácil achar esse
máximo -- igualar a derivada a zero na mão é impraticável.
 
 
Oi,
 
Cara, eu empaquei aí também...
Caso alguém aí tenha uma solução mais inusitada ficarei grato.
 
Abraços
 
Vinícius Meireles Aleixo







Re: [obm-l] Desigualdade de complexos

2005-02-23 Por tôpico Claudio Buffara
on 23.02.05 17:11, Fabio Niski at [EMAIL PROTECTED] wrote:

 Pessoal, travei nesse problema aqui. Alguem tem alguma sugestao/solucao?
 
 Sejam z[1], z[2] numeros complexos tais que |z[1]|  |z[2]|. Mostre que,
 para todo n = 2,
 
 n*(|z[2]/z[1]|)^(n-1)  |z[1]|/(z[1] - z[2])
 
 Obrigado
 
A desigualdade nao faz sentido pois o lado direito nao eh necessariamente
real.

[]s,
Claudio.

=
Instruções para entrar na lista, sair da lista e usar a lista em
http://www.mat.puc-rio.br/~nicolau/olimp/obm-l.html
=


[obm-l] 3 problemas em aberto

2005-02-22 Por tôpico Claudio Buffara
Restam, na lista, 3 problemas em aberto dentre aqueles propostos na ultima
semana. O primeiro, que eu propuz, eh de longe o mais facil. Para o segundo,
nao tive nenhuma ideia. Minha unica observacao eh que a reciproca (ABC
equilatero implica DEF equilatero) eh trivial. O terceiro dah pra fazer no
braco, mas obviamente o legal eh achar uma forma esperta de enumerar os
cortes. Eu pensei no numero de solucoes de x+y+z+w=8 com algumas restricoes
mas me enrolei.

Enfim, pessoal, vamos botar a caixola pra funcionar! Eh pra isso que essa
lista existe.

1) Sao dados n segmentos de reta (cada um de comprimento fixo mas todos
moveis), os quais, justapostos numa dada ordem, formam um n-gono convexo
inscritivel.
Prove que qualquer permutacao desses segmentos formarah um n-gono convexo
inscritivel e que todos os n-gonos assim formados tem a mesma area (e,
obviamente, o mesmo perimetro).


2) Seja um triangulo ABC. Marque os pontos D,E e F sobre os lados AB, BC e
CA tal que AD=BE=CF.
Prove que se o triangulo DEF for equilatero, entao ABC e' equilatero.


3) Dado um tabuleiro quadriculado de 4 x 4, com cada casa pintada de uma cor
distinta, deseja-se cortá-lo em dois pedaços de igual área mediante um só
corte, que siga os lados das casas do tabuleiro. De quantas maneiras se pode
fazer isto?

Obs. Os pedaços em que se divide o tabuleiro devem ser peças inteiras; não
devem ser desconectados pelo corte.

Resp: 70 maneiras


[]s,
Claudio.


=
Instruções para entrar na lista, sair da lista e usar a lista em
http://www.mat.puc-rio.br/~nicolau/olimp/obm-l.html
=


Re: [obm-l] 3 problemas em aberto

2005-02-22 Por tôpico Claudio Buffara
on 22.02.05 10:07, Domingos Jr. at [EMAIL PROTECTED] wrote:

 
 3) Dado um tabuleiro quadriculado de 4 x 4, com cada casa pintada de uma cor
 distinta, deseja-se cortá-lo em dois pedaços de igual área mediante um só
 corte, que siga os lados das casas do tabuleiro. De quantas maneiras se pode
 fazer isto?
 
 
 
 não sei se isso é equivalente ao número de soluções de
 x_1 + ... + x_4 = 8
 sujeito a  0 = x_i = 4
 
 onde x_i seria o número de quadrados abaixo do corte na i-ésima coluna.
 
 a minha dúvida é em relação ao um só corte... ie, x_1  0 e x_2 = 0 é
 um corte só? na minha opinião, não deveria ser, mas x_1 = 0 e 1  x_i 
 x_4 para i  1 sim.
 
Concordo. Por isso uma restricao deve ser 1 = x_2, x_3 = 3.

Essa foi justamente a ideia que eu tive. Por exemplo, a solucao (0,3,2,3)
representa um corte valido apesar de ter x1 = 0. Mas, se voce girar essa
solucao 90 graus, voce obterah uma outra igualmente valida que nao estah
incluida nas solucoes da equacao acima.

[]s,
Claudio.

 Obs. Os pedaços em que se divide o tabuleiro devem ser peças inteiras; não
 devem ser desconectados pelo corte.
 
 Resp: 70 maneiras
 
 
 
 =
 Instruções para entrar na lista, sair da lista e usar a lista em
 http://www.mat.puc-rio.br/~nicolau/olimp/obm-l.html
 =
 


=
Instruções para entrar na lista, sair da lista e usar a lista em
http://www.mat.puc-rio.br/~nicolau/olimp/obm-l.html
=


Re: [obm-l] Algoritmo do Calendário

2005-02-22 Por tôpico Claudio Buffara
Title: Re: [obm-l] Algoritmo do Calendário



Va ateh:
http://marauder.millersville.edu/~bikenaga/numth/numnote.html

O arquivo eh calendar.ps e estah em PostScript.

[]s,
Claudio.

on 22.02.05 11:34, Alan Pellejero at [EMAIL PROTECTED] wrote:

Pessoal da lista!
Fiquei sabendo da existência de um algoritmo matemático que trabalha com os dias do calendário.
Por exemplo, eu quero saber qual dia da semana caiu 22 de abril de 1872. Eu sei que o calendário gregoriando se repete de 400 em 400 anos e tal, já procurei no google, pedi auxílio a muita gente e até tentei montar esse 'algoritmo', entretanto, sem sucesso...Alguém poderia me ajudar?
Obrigado!

Yahoo! Acesso Grátis http://br.rd.yahoo.com/mail/taglines/*http://br.acesso.yahoo.com/ - Internet rápida e grátis. Instale o discador do Yahoo! agora. 






Re: [obm-l] 3 problemas em aberto

2005-02-22 Por tôpico Claudio Buffara
on 22.02.05 13:31, Fábio Dias Moreira at [EMAIL PROTECTED] wrote:

 [22/2/2005, [EMAIL PROTECTED]:
 Restam, na lista, 3 problemas em aberto dentre aqueles propostos na ultima
 semana. O primeiro, que eu propuz, eh de longe o mais facil. [...]
 
 1) Sao dados n segmentos de reta (cada um de comprimento fixo mas todos
 moveis), os quais, justapostos numa dada ordem, formam um n-gono convexo
 inscritivel.
 Prove que qualquer permutacao desses segmentos formarah um n-gono convexo
 inscritivel e que todos os n-gonos assim formados tem a mesma area (e,
 obviamente, o mesmo perimetro).
 
 Seja R o raio da circunferência circunscrita ao n-ágono, e O o centro
 desta circunferência. Se os comprimentos dos lados são l_1, l_2, ...,
 l_n e os ângulos associados de vértice O são a_1, a_2, ..., a_n, então
 a permutação l_p(1), l_p(2), ..., l_p(n) induz os ângulos a_p(1),
 a_p(2), ..., a_p(n). Além disso, como só estamos rearrumando os
 triângulos gerados por O e por cada lado, a área é preservada.
 
Isso mesmo. Com base nisso dah pra provar que, de todos os n-gonos inscritos
num dado circulo, o regular eh o de maior area.

 [...] O terceiro dah pra fazer no
 braco, mas obviamente o legal eh achar uma forma esperta de enumerar os
 cortes. Eu pensei no numero de solucoes de x+y+z+w=8 com algumas restricoes
 mas me enrolei.
 
 Se a sua idéia é a que eu estou pensando, o seguinte corte não parece
 ser representado por nenhuma solução:
 
 
 XXOO
 XOOX
 
 
 []s,

Precisamente onde eu empaquei. O problema eh aquele X na posicao (2,2) e nao
adianta girar o quadrado...

[]s,
Claudio.



=
Instruções para entrar na lista, sair da lista e usar a lista em
http://www.mat.puc-rio.br/~nicolau/olimp/obm-l.html
=


Re: [obm-l] Qual resposta ?

2005-02-21 Por tôpico Claudio Buffara
on 21.02.05 02:58, Bruno Bruno at [EMAIL PROTECTED] wrote:

 |  a   b | |  a   0 |  |  0   b | |  0   b |  | -b^2   0 |
 | -b   a |  -  |  0   a |  =  | -b   0 |  *  | -b   0 |  =  |  0  -b^2 |
 
 | -b^2   0 | | -b^2   0 |  |  0   0 |
 |  0  -b^2 |  -  |  0  -b^2 |  =  |  0   0 |
 
 Então, calculando o valor de ( A - aI )^2 + b^2 I, em que I éa matriz
 identidade de ordem dois, estamos... perdendo tempo!
 

Nem tanto. Estamos mostrando que existe um anel no qual a soma de dois
quadrados nao nulos eh igual a 0.

No caso, se supusermos que a e b sao numeros reais, o anel em questao serah
de fato um corpo isomorfo ao dos complexos, e a expressao matricial do
enunciado serah equivalente a expressao (a + bi - a)^2 + b^2.

[]s,
Claudio.

 
 On Mon, 21 Feb 2005 00:20:46 -0300, Bruno Bruno [EMAIL PROTECTED] wrote:
 
 
 Sejam a e b números reais e A a matriz
 |   a b |
 | - b a |
 Então, calculando o valor de ( A - aI )^2 + b^2 I, em que I é a matriz
 identidade de ordem dois, estamos..
 


=
Instruções para entrar na lista, sair da lista e usar a lista em
http://www.mat.puc-rio.br/~nicolau/olimp/obm-l.html
=


Re: [obm-l] desigualdade

2005-02-21 Por tôpico Claudio Buffara
Uma ideia: 
Chame o produto de A e defina B = (2/3)*(4/5)*...*(96/97)*(98/99)*(99/100).
Calcule A*B e compare A com B.
Isso resolve a desigualdade da direita.
Pra da esquerda, defina C = (1/2)*(2/3)*(4/5)*...*(96/97)*(98/99).

[]s,
Claudio.

on 20.02.05 22:33, Daniel Regufe at [EMAIL PROTECTED] wrote:

 Boa noite a todos ...
 
 Prove a desigualdade ...
 
 1/15(1/2)*(3/4)*(5/6)*...*(99/100)1/10
 
 agradeço
 []`Daniel Regufe
 


=
Instruções para entrar na lista, sair da lista e usar a lista em
http://www.mat.puc-rio.br/~nicolau/olimp/obm-l.html
=


[obm-l] Citacao do Newton

2005-02-21 Por tôpico Claudio Buffara
Title: Citacao do Newton



on 20.02.05 15:53, [EMAIL PROTECTED] at [EMAIL PROTECTED] wrote:

Se enxerguei mais longe foi por estar sentado aos ombros de gigantes. (Isaac Newton) 

Se nao me engano, a citacao correta eh:
Se enxerguei um pouco mais longe foi por estar em pe sobre os ombros de gigantes.

Em ingles: If I have seen a little farther than others it is because I have stood on the shoulders of giants.

Mas, na minha opiniao, o que ele deveria ter dito eh:
Se enxerguei um pouco mais longe foi porque inventei um telescopio melhor.





Re: [obm-l] TRIANGULO ISOSCELES

2005-02-20 Por tôpico Claudio Buffara
http://www.mathematik.uni-bielefeld.de/~sillke/PUZZLES/steiner-lehmus

on 19.02.05 22:27, Igor Oliveira at [EMAIL PROTECTED] wrote:

 Não consegui achar não.
 
 As duas bissetrizes têm comprimentos iguais.
 
 Se alguém puder resolver o problema, agradeço.
 
 Igor.
 
 Entre nos arquivos da lista e procure uma msg bem antiga do Eduardo Wagner
 com uma
 bela demonstracao disso. Ou entre no Google e digite Steiner-Lehmus proof.
 
 []s,
 Claudio.
 
 De:[EMAIL PROTECTED]
 
 Para:obm-l obm-l@mat.puc-rio.br
 
 Cópia:obm-l obm-l@mat.puc-rio.br
 
 Data:Thu, 17 Feb 2005 00:29:32 -0200
 
 Assunto:Re:[obm-l] TRIANGULO ISOSCELES
 
 
 
 O que vc quer dizer com 'duas bissetrizes iguais'
 por acaso seria duas bissetrizes de mesma medida?
 
 
 
 Peço ajuda para resolver o seguinte problema:
 
 Mostre que se um triângulo possui 2 bissetrizes iguais, então o triângulo é
 isósceles.
 
 OBRIGADo,
 
 IGOR
 
 =
 Instruções para entrar na lista, sair da lista e usar a lista em
 http://www.mat.puc-rio.br/~nicolau/olimp/obm-l.html
 =
 
 
 Atenciosamente,
 
 Osvaldo Mello Sponquiado
 Engenharia Elétrica, 2ºano
 UNESP - Ilha Solteira
 
 
 
 
 =
 Instruções para entrar na lista, sair da lista e usar a lista em
 http://www.mat.puc-rio.br/~nicolau/olimp/obm-l.html
 =
 


=
Instruções para entrar na lista, sair da lista e usar a lista em
http://www.mat.puc-rio.br/~nicolau/olimp/obm-l.html
=


Re: [obm-l] TRIANGULO ISOSCELES

2005-02-20 Por tôpico Claudio Buffara
E a solucao do Eduardo Wagner estah aqui:

http://www.mail-archive.com/obm-l@mat.puc-rio.br/msg09883.html



on 19.02.05 22:27, Igor Oliveira at [EMAIL PROTECTED] wrote:

 Não consegui achar não.
 
 As duas bissetrizes têm comprimentos iguais.
 
 Se alguém puder resolver o problema, agradeço.
 
 Igor.
 
 Entre nos arquivos da lista e procure uma msg bem antiga do Eduardo Wagner
 com uma
 bela demonstracao disso. Ou entre no Google e digite Steiner-Lehmus proof.
 
 []s,
 Claudio.
 
 De:[EMAIL PROTECTED]
 
 Para:obm-l obm-l@mat.puc-rio.br
 
 Cópia:obm-l obm-l@mat.puc-rio.br
 
 Data:Thu, 17 Feb 2005 00:29:32 -0200
 
 Assunto:Re:[obm-l] TRIANGULO ISOSCELES
 
 
 
 O que vc quer dizer com 'duas bissetrizes iguais'
 por acaso seria duas bissetrizes de mesma medida?
 
 
 
 Peço ajuda para resolver o seguinte problema:
 
 Mostre que se um triângulo possui 2 bissetrizes iguais, então o triângulo é
 isósceles.
 
 OBRIGADo,
 
 IGOR
 
 =
 Instruções para entrar na lista, sair da lista e usar a lista em
 http://www.mat.puc-rio.br/~nicolau/olimp/obm-l.html
 =
 
 
 Atenciosamente,
 
 Osvaldo Mello Sponquiado
 Engenharia Elétrica, 2ºano
 UNESP - Ilha Solteira
 
 
 
 
 =
 Instruções para entrar na lista, sair da lista e usar a lista em
 http://www.mat.puc-rio.br/~nicolau/olimp/obm-l.html
 =
 


=
Instruções para entrar na lista, sair da lista e usar a lista em
http://www.mat.puc-rio.br/~nicolau/olimp/obm-l.html
=


Re: [obm-l] 3 problemas

2005-02-19 Por tôpico Claudio Buffara
Bruno Bruno ([EMAIL PROTECTED]) escreveu:
 
 Estou com dificuldades com esses daqui:
 
 1) Qual o algarismo das unidades do número x = 1^1 + 2^2 + 3^3 + +
 n^n  ?
 

Minha solucao eh baseada no fato de que a sequencia n^n (mod 10) tem periodo
20. Mesmo assim, nao encontrei uma formula bonitinha.

No que se segue, as igualdades devem ser entendidas como congruencias mod
10.

0^k = 0 ==
final 0 gera a subsequencia 0, 0, 0, 0, 0, ... - periodo 1.

1^k = 1 ==
final 1 gera a subsequencia 1, 1, 1, 1, 1, ... - periodo 1.

2^(20k+2) = 4 e 2^(20k+12) = 6 ==
final 2 gera a subsequencia 4, 6, 4, 6, 4, ... - periodo 2.

3^(20k+3) = 7  e  3^(20k+13) ==
final 3 gera a subsequencia 7, 3, 7, 3, 7, ... - periodo 2.

4^(10k+4) ==
final 4 gera a subsequencia 6, 6, 6, 6, 6, ... - periodo 1.

5^k = 5 ==
final 5 gera a subsequencia 5, 5, 5, 5, 5, ... - periodo 1.

6^k = 6 ==
final 6 gera a subsequencia 6, 6, 6, 6, 6, ... - periodo 1.

7^(20k+7) = 3  e  7^(20k+17) = 7 ==
final 7 gera a subsequencia 3, 7, 3, 7, 3, ... - periodo 2.

8^(20k+8) = 6  e  8^(20k+18) = 4 ==
final 8 gera a subsequencia 6, 4, 6, 4, 6, ... - periodo 2.

9^(2k+1) = 1 ==
final 9 gera a subsequencia 1, 1, 1, 1, 1, ... - periodo 1.

***

n:
0  1  2  3  4  5  6  7  8  9 10 11 12 13 14 15 16 17 18 19
ultimo algarismo de n^n:
0  1  4  7  6  5  6  3  6  1  0  1  6  3  6  5  6  7  4  1
ultimo algarismo da soma parcial (1^1 + 2^2 + ... + k^k):
0  1  5  2  8  3  9  2  8  9  9  0  6  9  5  0  6  3  7  8


Logo, para n = 20m + r (m = 0 e 0 = n = 19),
o ultimo algarismo de 1^1 + 2^2 + ... + n^n serah igual ao ultimo algarismo
de A + B, onde: 

A = 8m (mod 10) 

e B eh dado por:
r:
0  1  2  3  4  5  6  7  8  9 10 11 12 13 14 15 16 17 18 19
B:
0  1  5  2  8  3  9  2  8  9  9  0  6  9  5  0  6  3  7  8

***

Por exemplo, se n = 1234 = 20*61 + 14, teremos:
A = 8*61 = 8*1 = 8  e  B = 5 ==
A + B = 13 = 3 ==
o ultimo algarismo de 1^1 + 2^2 + ... + 1234^1234 eh 3.
 
[]s,
Claudio.


=
Instruções para entrar na lista, sair da lista e usar a lista em
http://www.mat.puc-rio.br/~nicolau/olimp/obm-l.html
=


Re: [obm-l] poligono sem angulo conhecido, mas com area

2005-02-18 Por tôpico Claudio Buffara
Falando nisso, aqui vai um bonitinho e facil:

Sao dados n segmentos de reta os quais, justapostos numa dada ordem, formam
um n-gono convexo inscritivel.
Prove que qualquer permutacao desses segmentos formarah um n-gono convexo
inscritivel e que todos os n-gonos assim formados tem a mesma area (e,
obviamente, o mesmo perimetro).
Podemos relaxar as condicoes do n-gono original ser convexo e/ou
inscritivel?

[]s,
Claudio.

on 18.02.05 10:53, Nicolau C. Saldanha at [EMAIL PROTECTED] wrote:

 On Fri, Feb 18, 2005 at 11:13:23AM -0200, kandon wrote:
 Content-Description: Mail message body
 Existe uma maneira de desenhar um poligono de 5 lados sem angulos
 conhecidos? Apenas com os lados e a area?
 
 os lados sao 312, 252.16 , 13.50, 70 e 87.55 e a area eh 25000
 
 
 nao consigo pensar em nada para resolver isso..
 eh um terreno, ja tentei medir o angulo no local, mas com trena tem muito
 erro.. achei um angulo de aprox 93.16 graus entre e 70 e 312
 
 obrigado 
 
 Os 5 lados mais a área são dados insuficientes para determinar o pentágono.
 Pense nos 5 lados como 5 varetas conectadas pelas pontas:
 podemos escolher arbitrariamente dois ângulos consecutivos
 e colocar as três primeiras varetas nas posições desejadas
 e as duas últimas se acomodarão de maneira essencialmente única.
 Isto tudo é na situação genérica, supondo o pentágono convexo.
 Como temos 2 graus de liberdade, a área (1 dado numérico)
 é insuficiente.
 
 []s, N.
 =
 Instruções para entrar na lista, sair da lista e usar a lista em
 http://www.mat.puc-rio.br/~nicolau/olimp/obm-l.html
 =
 


=
Instruções para entrar na lista, sair da lista e usar a lista em
http://www.mat.puc-rio.br/~nicolau/olimp/obm-l.html
=


Re: [obm-l] Questão de dizimas(muito boa)

2005-02-04 Por tôpico Claudio Buffara
Title: Re: [obm-l] Questão de dizimas(muito boa)



mdc(97,10) = 1 == 
a expansao decimal de 1/97 nao tem parte nao periodica ==
1/97 = B/10^m + B/10^(2m) + ... = B/(10^m - 1), para algum m ==
97B = 10^m - 1 ==
-3B == -1 (mod 10) == 
B == 7 (mod 10) ==
o ultimo algarismo do periodo de 1/97 eh 7.

[]s,
Claudio.

on 04.02.05 12:13, Thiago at [EMAIL PROTECTED] wrote:

Uma boa questão: qual é o último algarismo do período gerado pela expansão da fração ? 

Internal Virus Database is out-of-date.
Checked by AVG Anti-Virus.
Version: 7.0.300 / Virus Database: 265.8.2 - Release Date: 28/1/2005








Re: [obm-l] Questão de dizimas(muito boa)

2005-02-04 Por tôpico Claudio Buffara
B/10^m + B/10^(2m) + ... eh a expansao decimal de 1/97, onde o periodo eh B:
um inteiro com m algarismos. Isso eh uma PG infinita com primeiro termo
B/10^m e razao 1/10^m. Logo a soma desa PG eh B/(10^m - 1).

mod 10 quer dizer que eu soh estou preocupado com o algarismo das unidades,
ou seja, o resto da divisao de B por 10.

[]s e bom carnaval a todos,
Claudio.



on 04.02.05 13:43, Alan Pellejero at [EMAIL PROTECTED] wrote:

 Cláudio, não entendi muito bem a parte
 1/97 = B/10^m + B/10^(2m) + ... = B/(10^m - 1), para
 algum m ..
 e o que significa (mod 10)?
 obrigado!
 alan
 
 
 --- Claudio Buffara [EMAIL PROTECTED]
 escreveu: 
 mdc(97,10) = 1 ==
 a expansao decimal de 1/97 nao tem parte nao
 periodica ==
 1/97 = B/10^m + B/10^(2m) + ... = B/(10^m - 1), para
 algum m ==
 97B = 10^m - 1 ==
 -3B == -1 (mod 10) ==
 B == 7 (mod 10) ==
 o ultimo algarismo do periodo de 1/97 eh 7.
 
 []s,
 Claudio.
 
 on 04.02.05 12:13, Thiago at [EMAIL PROTECTED]
 wrote:
 
 Uma boa questão: qual é o último algarismo do
 período gerado pela expansão
 da fração ? 
 
 Internal Virus Database is out-of-date.
 Checked by AVG Anti-Virus.
 Version: 7.0.300 / Virus Database: 265.8.2 - Release
 Date: 28/1/2005
 
 
 
 
 
 
 
 
 
 ___
 Yahoo! Acesso Grátis - Instale o discador do Yahoo! agora.
 http://br.acesso.yahoo.com/ - Internet rápida e grátis
 =
 Instruções para entrar na lista, sair da lista e usar a lista em
 http://www.mat.puc-rio.br/~nicolau/olimp/obm-l.html
 =
 


=
Instruções para entrar na lista, sair da lista e usar a lista em
http://www.mat.puc-rio.br/~nicolau/olimp/obm-l.html
=


Re: [obm-l] IV OLIMPÍADA DE MAIO

2005-01-31 Por tôpico Claudio Buffara
Title: Re: [obm-l] IV OLIMPÍADA DE MAIO



Muito justa a reclamacao do Fael. Assim, aqui vai a minha tentativa de solucao pro problema da Eureka 2 que ele mandou pra lista na semana passada e que ninguem respondeu.

Estou supondo que a peca eh movel e totalmente simetrica, de forma que pinturas que difiram umas das outras apenas por uma rotacao ou um flip sao consideradas indistinguiveis.

Chame os vertices de A, B e C e o centro de P.
Chame as cores de 1, 2, 3 e 4.

As varetas interiores podem ser pintadas de Binom(4,3) = 4 maneiras distintas.

Suponha, pra fixar ideias, que PA = 1, PB = 2 e PC = 3 (ou seja, PA foi pintada com a cor 1, etc...).

Caso 1: Um dos lados tem a cor 4.
Esse lado pode ser escolhido de 3 maneiras distintas.
Nesse caso, as cores dos outros dois lados ficam automaticamente determinadas (por exemplo, se AB = 4, entao soh pode ser BC = 1 e AC = 2).

Caso 2: Nenhum dos lados tem a cor 4.
Nesse caso, as cores tambem ficam automaticamente determinadas (AB = 3, BC = 1 e AC = 2).

Logo, o numero de pinturas distintas eh igual a 4*(3+1) = 16.

[]s,
Claudio.

on 27.01.05 06:40, [EMAIL PROTECTED] at [EMAIL PROTECTED] wrote:

Olá pessoal !

Problema 01 da IV OLIMPÍADA DE MAIO - prímeiro nível (Eureka 02, pag. 17): 

Com seis varetas se construiu uma peça como a da figura. As três varetas exteriores são iguais entre si. As três varetas interiores são iguais entre si. Deseja-se pintar cada vareta de uma cor só de modo que, em cada ponto de união, as três varetas que chegam tenham cores diferentes.As varetas só podem ser pintadas de azul, branco, vermelho ou verde. 
De quantas maneiras pode-se pintar a peça? 

Obs: A figura é bem simples ! Esboce um triângulo equilátero e una o centro desse triângulo com seus vértices. 



[]s, 
Rafael 

Se enxerguei mais longe foi por estar sentado aos ombros de gigantes. (Isaac Newton) 










Re: [obm-l] Eureka: Quantos quadrados ?

2005-01-31 Por tôpico Claudio Buffara
Title: Re: [obm-l] Eureka: Quantos quadrados ?



Seja n o numero de pecas que formam um quadrado de lado m. 
Entao, Area = m^2 = 6n == n = 6k^2 == m^2 = 36k^2 == m = 6k.

Para que tenhamos quadrados distintos, a cada quadrado deve corresponder um valor diferente de k.
Assim: 
k = 1 == m = 6, n = 6;
k = 2 == m = 12, n = 24;
...
k = p == m = 6p, n = 6p^2.

Numero total de pecas = 6*(1^2 + 2^2 + ... + p^2) = 1998 ==
p(p+1)(2p+1) = 1998 ==
p = 9 == 
podemos ter no maximo 9 quadrados diferentes ao mesmo tempo.

[]s,
Claudio.
 
on 01.02.05 01:47, [EMAIL PROTECTED] at [EMAIL PROTECTED] wrote:

Olá pessoal !


Têm-se 1998 peças retangulares de 2cm de altura e 3cm de comprimento e com elas se armam quadrados (sem superposições nem buracos). Qual é a maior quantidade de quadrados diferentes que se pode ter ao mesmo tempo?



[]s, 
Rafael 

Se enxerguei mais longe foi por estar sentado aos ombros de gigantes. (Isaac Newton) 






<    4   5   6   7   8   9   10   11   12   13   >